Sei sulla pagina 1di 83

Differentiol

& lntegrol
Colculus
UY
FELIC|ANOand

IreIreI
Visit For more Pdf's Books
Pdfbooksforum.com
^aifferential and Integf,al Calculus
Limits

n,general,if there rs a relation between two variablesx.and y such nxincrsn l.l


hat for each value of x, there corresponds a vairre of y, then y is
rid to be a function of x. Symbolically, this is written in the L. If f (x).= x2 - 4x, find (a) f (-5) (b) f (y' + 1) (c)
lrm* f(x + Ax) (d) f(x + 1).- f(x- L).
y = f (x) g* n

t
2, If y = , find x as,afunciioq of'y.
The function concep,tmay,be extended !o relations between
,or€ than two variables,Consider the equation" 3. If y = tan (x.+ 1), find r as a function y.
9f

z: f (xJ). ,4. Express the.distance D,traveled in t hr by a car whose speeO


is 60 km/hr.
his implies that z is determined'whe4.x and y are givefi and it ig
tstomary to saSrthat z is a function of x and y, Fof instance, the 5. Express the area A of an cquilateral triangle as a function cl
rlume of a right circular cylinder is a function of the altitude h its side x.
rd radius r of the base.that is, 6. The stiffness of a beam of nrctangutar cross section is propor-
tional to the breadth and tbe cube of,the depth, If the breadth
V=f(r,h):zrr2h. is 20 crn,. express the stiffhess as a function of the depth.
' I

It is irriportant that we he familiar with uhe functional nota: 7. A right circular cylincler, radius of base x, height y, is ins-
on In mathematics and the physical sci€nces functional no- sribed in a right circuiar cone, radius of base r and height h
tion plays a convenient and important 1rad. In the example Express y as functirin of x (r and h are constents).
rlow, we shall illushate how to set up a formula showing the
metional relation betwgen the variables. - f(0.
$. If f (x) = x2 + 1, finct f(x+h) ' ' h * o.
h
KAMPLE; The area of, a reetangle is 6 sg. in. Expre$s the pe. g
rimeter P qf the rectangle ,irs I function of the
I f f ( x ) = 3 x 2 - 4 x + 1 ;g 1 , 1 6 l ( ! * r y . ( 3 ) ' . 6 + o . .
h
length x of one side.
10. tf f (x) = +^ andB (x) = x2- 3, find r [s(x)] and'ufrt*t]
)LUTION: Since ct.: arge is 6.sq. in., then the length of the
other side !s perirneter is
f, and the 1.2 Limit of a Function
'Familiarity
with the limit concept is absolutely essentialfor
P=2(..*) a deeper understanding of the calculus. In this section, wO shall .
begin ourdiscussion of the limit of a function but we emphasize
that our treatrnent here will appeal more to our intqltion than
to rigor. And since our approart is a non-rigorous on", we there-
fore, expect you to traln lhiu itiea with eAse.
rThe notation
--., v = f (x, is due to the Swiss mai'rrematrcran Leonerd tiuter (1?(r?
E3).

Visit For more Pdf's Books


Pdfbooksforum.com
10 Differential and Integral Calculir Limits
11

The example above illustrates the fact that f(x) may have a
timit at a number c even though the varue f(a) of the fuirction is (x+312-t
5. ItII-t-
undefined. Moreover, it shows that the limit and value trr" x-O ZX
function are two different concepts "i

. . G / - - \ --^*",
f/ar\ 6. , lim
EXAMPLE Z. Evaluatelim '(fl if f(x) = x2 - Bx X-O

. x--+Z l(-ll vt r. X-'1


-:-
,. uifr
Solution: *- i ru/x* 3- Z
A straight substitution of x = 2 leads to the indeter-
_n
minate formf. Since f(x),= 1z - 3x, then f(2) = 4 - 6
= - t. Hence. 8.

-
,r^ f(x) f(4 _ lim
(x3-3x) - (--?)
x,+2 X-2 x- 2 x-2 9.
v2 - 3 x + 2
: hma x3 -8
x.* 2 x.--2 10. lim
x-2 xz -4
_ :_ (x-1)(x-21
: t tu--rr x-3
{iz x-2' 11. li-
x*3 ffi -.va-=
=lim(x-1)
x+2 L2lr,*(+-*s
r. r/xz - I
13. Ilm---'-
r-3 X-J

EXERCISEI.3
14.
Evaluateeachof the following:
Iim t^\'?*
7 sec zx
x 3 - 64 4
1. trln -
x-4x2-16
'xz
+2x-B 5- -ttr I-
2. r.
tun
15. a-6 SIDX - tanx
x*2 3x-G
- -1 - c o s 2 x
-3-13x+12 16. it:t 1 * cosx
3. lirn4-
x-3 X3-L4x+15
sinx sin 2x
4.
L7. ,. .
x--+o 1-cosx

Visit For more Pdf's Books


Pdfbooksforum.com
L2 Differential,axrd Integral Calcuius l.rmits Diffeiential and Intesat Oalcutus 13

1-+oo as ;+ o
18. lim ii4ll- x
If cosx
\_r

If f(x) : \E find In more compact form, we write


1 -oo
lim
19. x+o x

x 0.1000 0.0100 0.0010 0.0001 +0


f(9+ x) - f(9)
20. lim
x-.0 x
rt+)=* 10 1.00 1,000 10,000 +€

Iff(x)=x2-2x+3,find '
Bear in mind that * is not a number which results from di.
vision by zero. Recall that in the real number system, division by
f(x) - f(2)
Zt.' lim zero is not permissible. In fact, it can be argued that the statement
;;2 x*2
-
- tim f(x) = €
zz. rim f (T+ x2) f(2) x+a
x-o
rs not an equation at all since - does not represent a number. It
is merely used as a symbol to imply that the value of f(x) in-
creasesnumerically without bound as x approacheso. *
'l

1.5 InfinitY

Let f(x) be a funetion. If we can make f(x) as large as we 1.6 Limit at Indinity
please by making x close enough, but not equal, to a real number
c, then we describethis situation by writing' A functign f(x) may have a finite limit ev.en when the in-
dependent variable x becomes infinite. This statement ,,x b.ecomes
lim f(x) : € infinite" is customarily expressedin symbolism by ,.x -+ o'r.
x+a
consider again the function f(x) = +. It can be shown,(in-
where the symbol - is read "infinity". tuitively or formally) that ] limit (tt e ,rrr*uu,
x "ppro".n., "trr,rr*
= tn" table be- zero) as x inereaseswithout bound. That is,
In particular, consider the function f(x) |.
the
low shows that as x takes on values successlvelyapproaching
.'mber 0, th* ;;;; J+;ows largerandlarger.we savthatlx be' - '*Tht
symbol 6 is us€d for infinity if no particular referenceto sign is made. The
comes infinite as i approaches 0 and indicate this by writing symbols1 o (read "plus infinity") and - oo(read "minqs infinity") are used in some
bpoks in connection with statemcntsabor1tlimits. The symbol * oo is used to indicate
that f (x) becomespositively infinite (increaseswithout bound) while -6is used to
meanthat f (x) becomesnegativelyinfinite (decreases rvithoutbound).
Visit For more Pdf's Books
Pdfbooksforum.com
L4 DrlTerential and Inteffal Cate
16,

1
*O asx-->€ hom the examplesaboye, we intuitively feel that if n is ayrypoci-
i l&o number,then
we shall consider this fact as an additional theorem
on limits
in symbol, we wrife 1 =0
tim
!<co xn
lim 1 =o
Lg
\eF X
lhf"tr sjryn as a theoremin somebooHs.Note thet whenrri 1,
. Theuseof Lg is il*strated in ilre fonpwingerampres. tr have L9.
E X A M P L E I l. i m + = p n (!.1.!t A function f(x) =# may assumethe indeterminatefomr
;+61U 1-o\X X Xl
I when ir rplace$ b;y'}. However,the limit of f(x) as x be-
I
Fmes infinite may be definitc. To find this iimit we'first divide
= lim .fttl and D(x) by the highest power of x. Then we evalu"t" d";
-X+@ +. Hm+. lim 1
5' X X
hrit by useof Lg.
X-€ [+6
tbf t 3xz - 6
Bnupr,n: Evatuate lim
.1

x * 6 2 X s+ 5 x + g
=0
solution: the function assumesthe indeterminate fotm
EXAMPLE 2, lim L = 4 lim 1 fr when x is replaced by -: Dividirrg the numer_
ator and denominator by xs , we get
[+o,)12 x+o X2

+3x2 -6
e +x9x-t e
= 4- r i m & . 1 \ i.
lim -4 x 3 3 ilm
;:-\* vl x-@ 2x3+5x+3 x;6
2 +L +l
x2 v3
= 4lirn +. lim 1
oo X
X-+ *_*-* X 5+0-0
2+0+0
=0
by =2
EXAMPLE g. lim 1
---f = lrm (*tj-
XT xiPo wh EXERCISE1.4
llvduate each of the following.
= lrl i m 1l-i
.Lx-- TJ 0r9+4x3+S 8x-6
I lim 5. lib
x -.6 &8+?x-B +
r eo \/Tf,fT|9-
=0 by L9

Visit For more Pdf's Books


Pdfbooksforum.com
Limits L?
16 Differential and Infegral Calc

this'function is "unbroken" ovqr that interval. That is, the graph


2. lim
--') @
-el'z+ u-?= 6. lim
(2x - 1;z of f(x) can be drawn without lifting the pencil frorn the paper
X 13+8x+1 X+€
(seeFig. 1.1).
,. 4x +5 (x+2;r-(x-2
3. trm 7. lim EXAMPLE 1. The function f(x) - x2 is continuous at x : 2
11*oo12 + 1 X+OO x2 becauselilft, *t ==f(2) : Q,.In fact, it is continuous
x3 +x+2 for all firiiti values of x. The graph of the function,
4. lim lrW +T is shown in Fig. 1.1.
8. lim
x*ao x2-1 X-€ 6x+L
1.7 Continuity
'of
In Section 1.4, we imphasized that the limit and ualue
function are two different concepts. In fact, irt Section 1.2,w
we discussedthe mea,ningof lim f(x) = L, we deliberately i
x-+c
the actual value of f(x) at x = a. However, in Section 1.3, we
mention of the fact that the limit of a function f(x) as x *+ o
tum out to be just the value of f(x) at x = a That is, lim f(x) = fta)
x?rt
Now when this happens, we have an event of sonte mathematic
significance. The function f(x) is said to be continuous at x = c.
This leads to the following definition. -
DEFINITION 1.2 A function.f(x) is continuous at x = c
if lim f(x) = f(a). FIG. 1.1
x-+c

Note that the conclition lim f(x) = f(a) in the definition


above actually implies three namely
""frairl3nS,
(1) f(a) is defined.
(2) lirn f(x) = L exists, and

x.
x+d
(3) L = f(c)

If any of these conditions is not satisfied, then f(x) is saidrto


di*ontinuous at x = o.
r
A function f(x) is said to be continuous in an interual if'
is eontinuous for euerv ualue of x in the interual. The saph o

FIG. 1.2
$This def,rnitionwas formulated by the French mathematicianAugustin Lou
Cauchy(1789-1857). ,

Visit For more Pdf's Books


Pdfbooksforum.com
l8 Differential and Intesral Calculus '19
Limits

EXAMPLE 2. The function f(x) = * ir continuous at x = 3 'The lrrle


DEI.INITION T C v. = ,tr is,a h'orrsontal astmptote of
x
because 1= =-+ is, however, discon- tlre gfaph of f(x) if lim f('x) = b.
lim-+
x-;3x
f(3) i. -It .t(- €

tinuous at x = 0 since liri + = €. The graphof the zx -'€' then x = 3 is avertical


X x+o EXAMPLE 1. sitrce li^ ,?*;
function (seeFig. 1.2) contains a "break" at x = 0. x'+ 3X-d
asyrnptobe of the graph'of the function de'
4x
EXAMPLE 3. Is the tunction"f(x) = over the fined by f(x 1 = -31! *
x,Acontinuous
i n t e r v a0l ( x 5.5? 2.
Ex^at,tPl,s = 2 is a horizonthl asymppote of the graph
\
Answer: No, since at x = 2, f(2) is undefined. off(x)= #*- = 2.
tT"Stt-;ff3.
EXERCISE1.5 EXAMPLE3. Y = 0 ig a horizontd asYmPtote' of the
Find the value or values of x for which the function of f(x) o
3x
sinc€.lim;.'i=
t q- . 3x
0''
nuous.
ffi
4
E*AMPT'E
1.
3x
x-5
3.
5x+1
xz +4
5.
Z,!-a
ffi J# :'H,il^,ilP ::
2. - 3x,+2- , 4.
6x. x+3 From Definitions 1.3 and 1.4 and the exarnples abover we
x2-8x+15 6.
x2 -9 x3 :- 3x2 + 2; can mahe certain generalizations which would facilitate further ttre
'
process of findin[ ttre vertical and horizontal .asymptotes* of ttre
graph of the rational functitrn defined by the equat'ron
1.8 Asymptotes *' '
. . '= N ( x l
f(x) DG)+ o
Let f(x) = , D(x) + 0, be a rational function, i.€: N(x)
ffi,
##
and D(x) are polynomials. Suppose we wish to sketch the graph Sinee N(x) arld D(x) are,polynonrials, we may let
of f(x). A useful aid in sketching the graph of a.function is to find,
if there is any, the asymptote of its graph. The asymptote may be + &tX*- l , + +am_l + qn
N(x) = tolt
a uertical line (no slope).. a horkontal line 1zerc uopef of a non-
uertical line which slants upwald to the right (positive slope) or
D(x) = boxn + blxn-l + .i., + bn-l + bn
slants downward to 'the right (negative slopei. Ttre following
definitions are used to detennine the vertical and hori2ontal
where m and n are positiveintegersandao,Sl ,...arn urd brrbl ,
asymptotes.
bn are consbants,. We now fonnulate the following riil0sl for
DEFINITION 1.3 The line x = d is a vertical asymptote of the fOther propertbt of a corie lrdr u,iti
graph of f(x) if lim f(x) = *. :inlerccptr end symmetry erc arnrmcd
frmilirr to the studcnt.

Visit For more Pdf's Books


Pdfbooksforum.com
30 Differential and Intesal orr.ntiation of Algebraic Functions E1

-n
t+5tr12
= lim G i v e n V =+ z d , f i n d d V
At* o At s
d-;
14, divenS= 4rrr2,finddS
= 3(t+at)z -3t?
lim
At-* o At
t 6 . Givens - 2t + 1..find ds
3t- 4', At
= lim
6t'At + 3(at)z
At- o At
(hometricSignifrcance
"t #
= ^lim (6t + 661
o
At*
(lonsider the graph of y = f(x) shown in Fig.2.2.LetP(x,y)
Q(x + Ax, y + Ay) be any two points on this cunri. Line S
Intersects the curve at P and Q and having inclination d is
6t
the secant line of the cunre. Note that th6 slope of S is
EXERCISE 2.1
m= tan q = 4Y - f(x + a\rt - t(*)
ax Ax
-
Find the derivative by use of Definition 2.1 Y
f Y'f (x)
1. y-4*'-5x 7. y = V4x+3 I
I ,-Y+AY)

'2. v:*3 *2x " 8.t:ft I


I T

3, Y=4tE 9. v:
'
-&-
t/ 2x+L
,6
4. y:;
tr-2
1 0 . V = :. r-X%
- l'

5. y= {r
6. V=2-5x

11. Given,: y'f - z,rino


ff
AX
L2. GivenA=nf find {&
dr
FIG. 2.2

Visit For more Pdf's Books


Pdfbooksforum.com
38 Differential and Integal rrtiation of Algebraic Functions 39

(2x + l)s (0) - tZ (2x + LY Q) qv


4..
l ' l ,E 4. Fi nd y = (2x* 1;3 1ex_1)2.
(2x + 1)6 d; if

- 24 (2x + L\2 f{oltttion:


(2x + 1;e
- (2lr+1)3 tn* -1), + (4x-1), (2x+1)3
ll S * by Db
':24
=
(2x+ 1Y 1 2 x + 1 ) 3 . 2 ( 4 x - 1 ) ( 4 ) + ( 4 x - 1 ) r . 3 ( 2 x + L ) 2 (2) . by D?
, 2(Zx+1), (4x- r I +
2nd Solution: y= t' \rhx+!)-3
' why? [a(zx+ 1) B(4x-l)J
(2x+1)3
2(2x+1)2(4x-1) (20x+1)
dv bY
4
dx *(2x+1)-3 nxnRclsE2.2
dx rt eachof the following:
4(-g) (2x+1)- ".* (2x+1) by $
ox

= -12 (2x + 1)-o (Z) I y: 5x3 -4x2 + 3x-


.aa

= - ;24 (2x+1)-a
I v=Vi +f* V;

= .-2+. r y:.vF6_"-
( 2 x +1 ) a
a _

I y: t/2x-7
4
-_._-:-
3rd Solution: :
" (2x+1)3 t u=13xr-4x+1),

d.y= 4(-- gi _ tl y=.yffi


E-ffi fi{2x+1) ,, .. 4x-5
|
-
) i -

2x+1
=' - L 2 rzr
6if n r, = $.+1
--24 h"'.z
-
(r-.1tT ', v (2x+5)VE:T
lil \, (3x+4)2 (x-b),

Visit For more Pdf's Books


Pdfbooksforum.com
4l
Differential and Integrd I)ifferentiation of Algebraic Functions
40

l_L. v =/2*--il-
5 x +1 l
Find the values of x for which the derivative is zero.
[
\/ 26. Y= x3+4x2-3x-5
= 3x* 4
12. v -
2 7 . v : x4 -- 8x3 + 22x2 24x + 9
V 2x+5
/ \r
=
2 g ' v " 12x.u g;1
1,3. v =fr-s-\' 1
\.t,..n/ x-
29. Y' ;1#6
L4. y. = W - 4 x - '

L 5 . v : A(rrGI+ t)s F'ind the values of x given that

16. v =4 30. !=2x-3x'*dg=14


v6ffi*
2 dY: 1
31. y:x-' - x +3 a nrd
L7. Y= (4xfr)3 ??= a

g2. y=3x2+4xt*d*=11 -.-_t

Evaluate # at the sPecified value of x:

18. v- 6(Vx+,2)', x= I 2.5 The Chain Rule


t-
C e r t a i n f u n c t i o n s a r e f o r m e d o u t o f s i m p l e r f u n c t manner
i o n ' . b y 1are
19' Y= V6-Vx , x:4 of substitution. Functions which result in this
t)rocess '
t:allgdcomqosite functions'
2 0 . Y = :.3 + 4{l tt x:t i '
4 For a general discussion of composit-efunctions, consider the
2 L . Y = (2x'- 1)3 * ___:_ x= 2 functionstanaggivenbyy=f(ujandu=g(x)respeqtively.We
Vsx- z have here a situarion in which y depends on q and u in tum de:
u = g (x) in.J =-'f
'end,s on x. To eliminate u, we simply substituh
symbolically
Find the slope of the tangent to the curve at the glven iu) and thereby obtain a new function h expressed
:n the form
22. Y=?-x2+4x3 ,(*1,2)

23. V=x+2x:t , (2,3)


( 24. V:3x?*+, (2,10)

2,5.y- {F, (B,zts)

Visit For more Pdf's Books


Pdfbooksforum.com
44
Differential and Integral
Caleulus nnt.iation of Algebraie Functions
45
The functionl f
g ar,e said to be inuerse
tinguish betw.en.rTaani functions. To dis_
;-;" E D''rrr
shall ca' ri1t"
tl dtrect function and dv 1'
g the inuetse iunction -4=
"a' &:spsv
Let us now focus ourattention,to the problem EXERCISEZ"S
derivativeof v.witl.airodto"*"oi#,"Tinction of finding the
form x : gd). firis'is a wrirtenin the lhn (ihain Rule to find
u.compifrhedby and expressthe final answer in
inuersefunction ntte whichwe state-aslofioi* us)ngthe so called of x.
$

INVERSE PUNCTION RULE: I y:u2+u u- 2x] L

If y is a differentiable functian t y:1FT , U:418


y : f(x), then fts rnvense of x defined by
function detineo by x : g(y)
is a differentrabie function of y .3
and v - (u- 4rT , lr= xz + 4
Dl1: dv
--+ = v - (2u- 2r+ , u= 4x3+ 1
dx dx/dy

Note that D11 ,clearly shows that the \filTry , lr = 4x- Z


with respect to. (dv/dx) ana the rate rate of change of y
2u
pqct to y (dx/dv)_x
recinrocals.
oi
"t
aog of x with res-
ffi ' u=xz
3re
tiue of the inuerse functioi It arso says that the deriva-,
is equart" ti" reetprocarof the de-
riuattueof the direct sunction Td
prbof oinrl is givenberow. , ll: \E
. Proof of D11: Let y = tlx) anOx : g(V) be invers.e |ftr lnverseFunction RuIe io find gI .
tions. Then v is a f'nculn func_
";;lir-uJunction of y. By DL0, dx
qI ft, x=y*yr*yt
= jy. d"
dy dx dy
*=.,F+ f
and r =dy.di
t--
dx dy |0 (4 - 3v)*

or dx- I ll ZGy + 1)'


dx dx/dy
l2 *= 6 ,
EXAMPLE.If x = y3 _ 4yr,, find (3y+ 1)2
ff.

'|{,.€9'
t3 x-
Solution: Since x : Vr _ 4y,
, then
$}-Byr-8y
and by D11,

Visit For more Pdf's Books


Pdfbooksforum.com
46 Differential and Integral
,.rrtrirtion of Algebraic Functions 47

# : r"'(x):
Y"'= Div
2.7 Higher Derivatives
3rd derivative
Recall that,from the equation ! - f(x), wc get by di
tiation the equation : 1(a) D; y
$# rx)= y(4) : 4th derivative
g_ rf (x)
dx-
dny : 1n) (x) - y(n) =,It I u nth derivative
The derivativ. $f, or f'(x) of the function f is a nu{nber
pends op x. Hence f is itqelf a function of"x and may be di 1.,t.. t,hat parenthesis are used in y(n) ani fn) (*). The
prl r' ( n) rs used to distinguish rt trom the symbol yn. Re-
tiated again with regpeet to x" Thig proce,ssis represented
$rrt yn indicates the nth pou)er of y -- f (x) while the pre-
lically by any of the followihg notations:
n,trrt.i<rny(n) indicates the nth deriuatiueof V: f (x). The.
frirrr)r) holds for the symbol 1(n) 1xy.
, d , 1 ! X\ , - d 1 y _
m l..d,r':- ,l"t
IfY= x4-2x3+bx2- 4, then
gy= '-
*rf'(x)l= f"(x) Vt : 4x3 6x2 * 1Ox
dx

$ rr,l: yf, s3:


dx2
y"=12x2-12x+10
-

Dx (Dxy): D3,y
s3 Y"' : 24x- 12
dx3
.
d a Tv:
If we refer to4* as the first derivativeof y = r(4), y ( 4 ): 2 4
dx*
d2v .dx
shall refer t" (read ..d ffquaredy clx squared'l)as the
fri d5v -
deriuatiuesf y + f(x). The operator da inOicatesthat Y ' y(s)= 0
at dxs
fr;
t
is to be differentiatedtwice,
EXERCISE2.4
Further differentiations give us the derivatives of
It*l tlr. second and third derivative of each of the following:
higher than 2. These derivatives'are defined and denoted
lows*:
I r xs+3x'2+4x
( - practice,the
.. F lytsPoly" (ready doubleprime)andy" (ready rriple
usedin placeofy(z/616y(r/ respectively.
Similarly,
f'(x) andf' (x)areused
d
4?Jt-l *o 63)-1x)respectively.Thesymbor y" shouldnotbeusedin placeof
da)rxl.

Visit For more Pdf's Books


Pdfbooksforum.com
48 Differential and Integral I )ifferentiation of Algebraic Functions
49

4. - 3{- In this form, y is said to be an erplicit function* of x. For €xarn:


"= x*l ple, in the equation y. : x2 + 4x * 3, y is an explicit function
r > fx .
5. y :_(x + 5)' If y is a function of,x but is not expressedexplicifly in terms
/ tT - * ? ) ' \ ' trf x, then y is said to be artimplicit fuiction of x. In
6. y=(a each of the
r'(luations below, y is an implicit funetion of x.

7. v- = 1+\E-
L. x2+4xy*4y2:O
l&--
x 2:. Z- (1 *x)lny= 9
8. y=
VFT 3. yz =4x2+9

9. x2
-! = 4. l&-fy * xy : 2l
x*l
r0. If y = \tE, find f' (g) and f" (g). 5. ex -- cos y

11. If y : x5 , find ya and ye). Equationl (3), (a) and (b) can be written in the form grven
by equations (1) and (z), i.e., the right mernber of the equation -
is zero. Then, in general, an implieit function may be represented.
L2. Find ttre point on the cunre y = x3 * Bx for
by the equation
y' = y".
E (2.3) f (x, V) = 0
13. How fast does the slope of the currr€ 5l : ( x 2 + x
change at the point where x = Z? An implicit function given in the form E (z.g) can be conyerted
t.o the form y : f(x). For iUstance,the explicit form of the'equa-
L4. Find the rate of charrge of the slope of the tion(3)aboveisy:JWwhilethatofequation(5)isy-
y=x3-1at(2,71. Arccos ex. The reader is urged to obtain the explicit forms of '
equations (1), (2) and (4). How-ever,there are implicit functions
2.8 hnplicit Differentiation which are quite difficult (and may be quite imposible) to convert
to their corresponding expricit forms.-Thus findine g{ from an
In the preceding sections, we have been concemed implicit relationship between x and y is of particular*iinportance
with functions defined by the equation in those cas{eswhere it is difficult (if not impossible) to obtain
:rn explicit solution for y in terms of x.
y = f(x). A
To find ff or y' of an impricit .function, vr€ differentiate
both sides of the equation with respect to x and then solve for
,* ot y'. The pro."r, inqolved is called implieit differentiation.

+If from y = f (x),


we solve for x^in terms of V,- fe pet the fo# x = g (y). In
lhis latter form, x is said to be an explicit function :
of v.

Visit For more Pdf's Books


Pdfbooksforum.com
50 Differential and Integal Catculus
Differentiation. of Algebraic .Functions 5,1

EXAMPLE 1. Find -gv if y' = 4x2 + 9,r


dx
since x2 + y2 = 4
Arl
Solutionl
# (t'): # 14x?
+ e)
Note that y" catl also be oirLaittedwithout solving fot Y'
;n tcrms oi r a.rciy. Thai, is, stantng wiln zx + Zyy -- U ol'
2Y#: 8x* o
x*yy'-0
dy- 4x
dxy we can clifferent,iater-mplicity again to obtain

1+W"ry'y'-0
EXAMPLE 2. Find y' if x2 + 4xy+ 4Y' : O. 1+yy"+(y')t =Q

Solution: 2x* 4xy,.lr*,r..r??, Solving for 51",we get


- (y')t
(4xf Syly' y"=
v'
' i

-x
EXAMPLE 3. Find y" ifx2 + Yt = 4.. Substituting ft 3-
in the equation above and simplifying;"we
v
Solution: Differentiating with respect to x, we have ,rt4
get y
2x*ZYy':0
r-X EXEncrsE2'5
i-- =-
v . dv . . i. ..;-
Find Te bv implicit differentiation
clx
Differentiating further with respect to xo

Y(-1):(-x)Y' 1. x3 + y3 * 6xy,= 6
Y":
y2
2. x2+xy2+,y2=L
x
-r
= ' y 2 .(f)
v 3. /EE * xy.= 21

4. /I+\,6= VA-
-y--x-
7,

5: b? X2 * A.2y2 ,= ,Al b2
yr-
- (x'+y') 6" (x" y')r = (x+ )"
v' 7. y =4(xz +y')

Visit For more Pdf's Books


Pdfbooksforum.com
52 Differential and Integral Calculus
l,\1r1,61P

e' v2 =
R (3SA
" \2*-3)
9. y2-3x*2y:0

10. Some Applications


of the Derivative
Find y" in each of the following: 'f
lrr, rieriuatiueis a powerful. toel in the'solution of many
rr:; rr)science,engineering,geometry qnd economics. Among
11. x y = 32 I'r,,lrlcms which Srouwill find not oniy useful but also quitb
2 aa
I rrr11lre those situations which call for maxi$izing or mini'
L2. xT + YT-:37 rr lunction. For instance, a manufacturer is interested in
' urtl his cost of production. An engineer may want to'
13. y2--16x-o rrr,, t,he dimensions of the strongest rectangular beam that
r'ul l'rom a circular log of known diameter. A farmer may
74. x2-2xy*3y2*4 t,, f rrrrt the area of the largest rectangular fieldwhich he can
rvrt,h a given amount of fence. We shall find that the
15. 41l+9yr-36 is a very useful aid in soiving such types of problems.
r:hapter will introduce the students to some applica-
Find the slope,of the curye at the glven point. tltrl rlerivative. -
a

16. 2x3 + 2yt - 9xy at, (2 ,1) lrlr,rtions of Tangentsand Normals


lr, ,.,','tion 2.3, we have seenthat ihe derivative of a function
L7. Yt : x2 -- 1 at (3,2) ftr rrrt.rpreted as the slope of the tangent to the graph of the

18. x2 + +rrfiu + y, 25 at (4, Ll

19. x3 + x2y * V3 : I at (- I,Z)

JI 20. r/Fx+ ffi=5 atl},l)


t
2t. A circle is drawn with its center at (8, 0) and with radius r
such that the circle cuts the eltipse x2 * 4y, : 16 at right
angles.Find the radius of the circle.
22. The vertex of the parabola y2 : 8x is the center of an el-
(, lipse. The focus of the parabola is an end of the minor
axis of the ellipse, and the parabola and ellipse intersect
- at right angles. Find the equation of the ellipse.
' L'l,r r\r)H application of the derivative is a technique called Newton's meth,qd.
' I r n,rltlt:sonc to find the zeros of a function as accurately as desired. This is
, I r n I l r r sl l o o k _

Visit For more Pdf's Books


Pdfbooksforum.com
Differential and Intceral i3r1,ls1'61ions
of the Derivative 5b

funetion. In Fig. 3.1, the line T is the tangentto the curre y


tsy E(3.2), the equation of the normal is
Pr (xr , Vr ). The other line N perpendicularto T at P1 is
"l
the.normal to the curve.
y- 8:#, (x- 2)
tf v - f{x) is differentiable at x, . i.e., f (x, ) ex x+72y-98 =Q
we may formulate t6e tottowing ctet'initions about the
normal tothe currc y : f (x).

DEFINIIION 3.1 The tangent to the curve V. = f(x) EXERCISE3.1


(1, Yr ) is the line through P1 with
f' (xr ). lhr' ,'quationsof the tangentand normal to the graphof the
hrrr,'tionat the given point.
DEFINITTON3.2 The normal bs the cunre y = f(x)
(x' Vr ) is the line through Pr and 3x2-2x+L
I v , (2,9)
dicular to the tangent at P, .
I y-1+h/F, (4,?)
The equation of the tangent is given by the poin
of the equation of a straight line in analytic geometry, that ir v =xfi-iT
I , (b, 10)
-3
E(3.1) Y-Vr=m(x-xr) a y: : (2,2)
4-=,

It Y - x . , 2- , (1,2)

lfr,.rr. will the tangent to y - \E at (1, 2) crossthe x-axis?

At what point on the curve xy2 = 6 will the normal pass


lhr,rrrghthe origin?.
where as defined above m = f'(xr ).
Itnrl the area of the triangle formed by the coordinate axes
fn,l llre tangent to xy = 5 at (1, 5).
EXAMPLE: Find the equations of the tangent and
the curve Y = x3 at the Point (2, 8). Jrrrrl the area of the triangle bounded by the coordinate
.r,.r; ;rnd the tangent to y - I' at the point \2, +).
Solution: The point of tangency is (2, 8). Henee xl =
l'.rr,l the area of the triangle formed by the x-axis, the
Vr = 8. Sincey' = 3x2, then m = B(2)2 = 12. r r r r g r , nat n d n o r m a l t o x y : 4 a l ( 2 , 2 ) .
By E(3.i), the equation of the tangent is '
) 'rrrl the tangent to x2 * y' = 5 and parallel to 2x-y 4'
y- 8 = 12(x- 2) -
or 12x-y-_16=0 l rrrr|:r norrnalof slope the curvey2 :2x3
*to

Visit For more Pdf's Books


Pdfbooksforum.com
56 Differential and Integral some Applications of the Derivative
57

1 3 . Show that the iangent wittr slope m to y2 : 4ax is


-v = mx +'-L'
'm

3.2 Angle Between Two Curves


y=fr (x )

Pe(xo,Yo;

ottrer, then such'tangents form a pair of acuie anglesand a


obtuse angles. The acute and obtuse arnglesare supplemen

For a general discussionof rhis concept, consicl.er the


y - ft (x) and V == (x) which intersecr at a point Po (x
.f2
as shown in Fie. 3.2r Let ft anci$1 be the inclinations
tangents Tl and T2 at P,. respectively. Let d be the ao
tween these tangents. Then, by clefinition, 6 is also the
between the curves. It can easiiy be phown that
{, ft ,
are related by the equation
FIG. 3.2
6 =*, -+,
-
(1)
The sign of tan d in (4) is positive or negative depending
Then taking the tangent of both sidesof (1), we get ttr()n the values,ot
->old *z or on the order in whibh m, u.td
fn, rrreused. If tan T,
0 0, th5n 0 is acute and if tan 0 < 0, then
tan @= tan tAz --fi ) (2't 'l' rs obtuse. In most books, it
is customary to find only the
rrr'rr[cangle of interection between the curves. The same is tnre
rfr tlris book. Since tan Q
- tan#, t lr. absolute value symbol in the lrght member qf (4). Thus
tan o' = ttn4 (3) ,rrr fird formula would be
or 1+tanffLanfl1
tllz - [r
Let m, and m2 be the slopes of T, and T2 respectively. E (3.3) tanS=
= tan ff and m, = tan€7. Substituting
1 ?Glm,
Tf thlse in eq
(3) above,we obtain
* lr.rc the values of m, are given by the derivativesof the
t r r n < ' f i o nast P o ( * 0 , V o ) .Td Tz
That is,
h2 -11r-
' =
tand (4)
1*rDzm,
ffi,=
* [r,r^l] u,r"

)f1
ffiz=
u l f12 r * r l ^ +4tD r
'{* | \'\,' |
uAJo

Visit For more Pdf's Books


Pdfbooksforum.com
58 Differential anC Integral Calculus Som€ Applications of the Derivative 59

EXArvrpLE: interseetion the


between
il*Ji: :.tT ffitill
Solution: Solving the given equations simultaneously, w€
Set X : 4 and y : 2. Hence the point of inter-
section is Po (xo, Vo) = (4,2\ as shown in Fig.
3.3.

Differentiating the first equation x2 : 8y, we get

dvx
t=T= mr

Similarly, differentiating the second equation xy = 8, we


get

dy -8 -xy -v
S-Fl g=
, += JII^
ox x, x" x.

Thereforeat the point (4,2), we have

ml -
4
_ - l
-1
4 FIG. 3.3

q =- 2 - - I EXERCISE3;2
42
Find the acute anglebetweenttre given cules.
Thenby E (3.3),.weobtain
lrl
I
t-+-r
tanQ=l-#l
I 2 .x2+y2:5andy2:4x+8
lt .1l + ( - + ) ( 1 ) l
3, x'y * 4a'y - SUi andx2 = 4ay
=l-31=
r 4. =9xqpd3x2=. 4y
,2y'
5. x2y* 4y:8andxzy:4
Hence, 0 =Arctan3
6. xy = 18 antl y2 : 12x
Q=71"34'

Visit For more Pdf's Books


Pdfbooksforum.com
62 Dif,ferential and Integral ,ll,plrcationsof the Derivative 63

12: [1,3'!

I.:. [3r'- )
EXERCISE 3.3

I2 rrr[.rval or intenrals where the function is jncreasingand


rl rr,rlt'creasing.

I f(x; : 2x3+3x2-36x

I f(x): xt-3x+3

I f 1 x ;: (*t - 9)'

I l(x) : 3x2-6x*9
In each of these subintervals, f(x) is either
sing or decreasing. In the subinterval I' if
(choose any convenitlpt value of x within I
I f(x): x3-6x2+4
binterval), we see that f'(x) ) q and
t f (x) ==xo * 8x3 + 22x2 - 24x+ L2
ouently f(x) is increasing in r 'It t
f'{x) < o and f(x) is deereasrngin Ir. If : ' whether the function is increasing or decreasing in the
f ' ( x ) > o and f(x) is increasingrri :Ir. The
|lnrl'rrl. -
of the function is shown in Fi,t. 3.6
v I f11;: \6 (1,4)

tt I f(x): xt- 4, [- r, t]
I I f1x;= 6x- x2, t- l, al
a.
i
I
I
l.r flx; = x8_ 4x3+ 2x,
Lr
'_l
,I It f(x) - 6x* 3x2-
,l
1 y- xl6*+9x+l
sx',
:*'[l'l
2)
| +
a It f( x ) - = 3 x -
t

2
Inrrrrnand Minimurn Values of a Function

l s.r,t' r,s(' y : f(x) is a f unct ion which is cont inuous f or . all

C *i,r ,l,rnirin. Let the graph of this function be represented by


ts.r't' 'ht>wn in Fig. 3.4 in the preceding section. For the
dl {ftt ,,l fhc curve. we observe that
FIG. 3.6
lletweenAandB. f'(x))o

Visit For more Pdf's Books


Pdfbooksforum.com
66 Differential and Integal Calculus
Some Applications of the Derivative 61

Next, we consider x = 3. Following the pro-


cedureabove, we find that . EXERCISE 3.4

: (-) orY' ( 0 Find the value or values of x for which the given function has a
when x ( 8. y' : (+) (-)
rrraximum or a minirnum value
when *18, y'= (+)(+): (+)orY')0
1. v=8x3-9x2+1.
This. satisfies (2) of FDT and therefore, y is a -4x2 * 4x
2. v:x3
minimu'matx- 3.
(b) Substituting x :'1 in y =-x3 - 6x2 * 9x - 3, 3. y=4x-r *x
we get y = 1 which is the maximurn value of the
function. Likewise, substituting x = 3, we get 4- Y=xa
y = - g. ttris is the minimum value of the func-
tion. 5 ' v = x 3 - 3x2 + 3 x
(c) Therefore, the mucimum point is (1, 1) and the
minimum point is (3, - S). The graph of the x2+ 1
6. y:
function is shown in Fig. 3.?.
7. y = x 2 ( x - 1 ) 2
v -
8. 4y : 3xa ' 16xt + 24 xz

i^I I 3y=x3+3x2-gx+g

-
I 1 0 . o,- x2 4x + 5-
x-2
31
't 11. Y:x3-3x2+3
e
'1 12. V=x3-6x2+9x*3
fl
at
-oi 31t
___)x 13. V:2x3--9x2 +72x+4

,ll 14. y- (x-2)o

"l
-37 3.6 Significance of the Second Derivative
(3.-3)

In section 3.4, it was shown how the sign of the first deriva-
tive or y' of a function y : f(x) determines *tt"tfr"i the function
tr n maximum or a minimum at a critical value of x in a given in-
FIG.3?
lr

Visit For more Pdf's Books


Pdfbooksforum.com
68 Diffetential and lnterral Calculus \s'1'lr:ationsof the Derivative 69

we'recall that by definition


terval. Now we shal:t show how the sign of the second derivative
or y" may be used for the same purpose. d r y :*(*)
E,
consider again graph in Fig. s.4 but this time with
q,rr\r R
!h. '- the slope of the curue or the tangent at any point, then
rvr.sD y ru added asshown
.rnus(r as sno'wn in rigl'.8. For the
the part
part ABQ,
ll1l5-3.I9 m .(-lg.
observe that the curve arways lies berow its
lr.u. r.or ABa, we
;; 11t;
tangent. It is cus
tornary to say, in this case, that the cun,e i,
(as seen from below). It is clearly seen, "inior"-'ii*lr*ora
likewise, that for the
# fitsrone)
part'QcR, the curve always lies above
its tangent. Here, we say ,,t y" measuresthe rate of change of the slope of a curye-
that the eurve is cbneaue upward (as seen from.a-bove). l.l
Irrt nlong ABQ of the cuwe in Fig. 3.8, # (slope) is nega-
d
nlong QCR, # (ttope) is positive. Hence, we conclude

'l'he graph
of y = f(x) is concave upward if y" ) 0 ano
('()ncavedownward if y" < 0.
v
I n ;r maximum and C is a minimum point, then we
'lrrrk'that

-
y =f ( x ) I'he graph of y = f(x) is concave downward at a maxi-
rnum point and concave upward at a minimum point.

r (i ) and (i i ), we may fopmulate a test for determining


,r frrnction y = f(x) is a maximum or a minimum at a cri-
ol'x.

I Derivative Test (SDT)


=
l f r l'lrc function y : f(x) is a maximum at N Q
u r f t ' '( c ) : 0 a n df " (c)<o-
a
'l'he'
f lr function y : f(x) is a minimurn at x = 4
n ' -)x (c)> o.
\rl rl f'(c)=0andf"
O
lN'.f. tlrirt if f" (q) == 0 or if f" (c) does not exist, then SDT
--
lrl ,rrr<lerthis particular situatton' we may use FDT.
-7'

tl."
ft l',l: F i n d t h e value of x for which the function
d
y-x3 - 6x2 * 9x - 3 is a ma:rimum or a mi- I
FIG. 3.8
d
nimum. r
Visit For more Pdf's Books
Pdfbooksforum.com
llr1rl 11',r1ions
of the Derivative 77
70 Differential and Integral

Solution: Since y : x3 - 6x2 * 9x- 3 r,rrrf s. statement (1) says that y = fhd has a peint af in-
- i2x* 9 = 3(x -- 1) .t il if the second deriuatiue is zero ot s : a and the
then Y' : 3x2 jttriutitte changes slgn as the ualue eI x inereasesthrough
and Y":6x-1.2 rrt (2) states that if the secondderiuatibeis zero buuhe
lrlr' rs not equal to zero a.t x : a- then ! : f(x l hos o
talltctionatx=g
Setting Y' : 0, we ggt x = 1 a n d x : 3 .
note that
,I ll: Find the values ot x for which the curve of
when x:1rV" <0 v:x4-4x3has points of inflection.
when x:3 ry" >0
fott: Y' : 4x3 - L2x2 : 4t<2(x - 3)
Therefore, the function is a maximum - 2 4 x - * L 2 x( x - 2 )
and a minimum at x : 3. The res/ults y": !2x,
with the results in the example grven
3.4 krtting Y" : 0, we get x = 0 andx - 2. Now we test
lltrttt'valUes.
'A
point where the sense of concavity changes ir
point of inflection. For instance, in Fig. 3.8, consider (rl B y s t a t e m e n (t 1 )
-
The curve to the left of Q is concaae downward
right, the curve rs concaue upward. Hence Q is a pbmt Forx: 0 : when x( 0 ,V" ) 0
tion. Like the maximum and minimum points, the. wherr x)0,V"(0
flection is an essential feature of a curve when one is
graph of a function. Forx:22 when x(2,V"(9

It can be shown that if v = f(x) has a point of when x)2,V")0


x = n, then f" (a\: 0 or f " (o) does not exist*. To
the curve of the function has a point of inflection at Since the sign of y" changes in either case, then
f
value, we may use any of the following tests: the curve of the function has points of inflection
atx:0andx--2.
a POINT,OF TNFLECTTON TESTS err)
a I l,I By Statement ( 2)
(1) If f'!(a) = 0 and if f'(x) >a O f o r x ( a a n d
' Differentiating further y", we get
for x ) e, then y : f(s; has a point of i
a x=c. y"'=24x-24-24(x-1).
When x - 0 , y"' :-24*O
.) (2) 1f 'f"(c) = 0 and if fo'(c) * 0, then y.= f(x) hal
l'
of inflection at Y =- Q, When x-2, !"'=24*O
Since y"' + O, then the curye of the function has
tl
Jrointsof inflection at such values of x.
tThis fact is statedas a theorem in somebooks on calculus.
)l

Visit For more Pdf's Books


Pdfbooksforum.com
72 Differential and Integal liorne Applications of the Derivative 73

EXERCISE 3.5 Itrns in q surprisingly wide variety of problems in science, engl-


nrx.ring, geometry, economics, and other disciplines concemed
Find the value (or values) of x for which the curye of with ma:rima and minima. These problems, whether they are of
function has a point of inflection. grnu'tical importance or simply of theoretical rnterest, are often
rlfr'rred to as "maJc-min" problems. In solving problems of this
1. y:(x-1)o(*-G) I vgrt',no general rule applicable in all easescan be given. However,
thr. readermay find t-hefollowing stepspossibly helpful:
2' v:2x3.- 3x2-36x+25
1. Draw a tigure whenever necessaryand denote the variable
quantities by x,;ro z, etc.
3' v= 3xa'- 4x3+1
2. Identify the quantity to be maxirnized or minimized
4. y:x4-4x3+4x2 and express it in terms of other variable quantities. If
possible, express this quaritity in terms of one indepen-
5. v=3xs-Lbxa+20x3+3 dent variable.
3. Find the first derivative of the function and set it to
Find the ma:rimum, minimum or inflection point'of each zero. (why? ) The roots of the resulting equation are the
gtven curves. Sketch the graph. critical numbers which will give the desired maximum or
minimum value 'of the function. (Note: The critical
6. v:x3-3x2+4 number which gives a ma:rimum or a minimum value'may -
be verified by SD?. However, in practice, the desirecl-
7. 4Y : 3xa - 16x3 + 24x2 value can be selectedat once oy inspection.)
} , , \ A M P L E1 : A long strip of tin 30 cm wide is to be made
8. 3y=x3+3x2-9x+3
into a gutter with' rectangular cross sectiori by
y=
2 turning up equal widths along the edges. Find
9. v= 4
x1 the depth of the gutterivyhich yields the greatest
carrying capacity
11-. Y = - 6 x Solution: Let x - depth of the grtter (Fig. 3.9)
x2+3
V : base of the rer;tangularcross section
11. Y: 5x - xs A = area of the re,:tangular cross section
n
- L2. Y= **'-$*'
1 3 v' - .4* -
xt*4
,
rD 3.6 Applications of Maxima and Minima
30 cm
fl . The methods of determining the maximum or
value of a function in the pneceding section find r4any
FtG.3.9

Visit For more Pdf's Books


Pdfbooksforum.com
74 Differential and Integral Calculus I lrfferential arrd Integral Cglculue
76
To ,insure ther greatestcalrying capacity, we must make
the area of the erosssection gre"t as-possible.That is,
we maximizeA. Thus "" The volume.of the cylindrical tank is
V = zrt'h (2)
A= xy (1)
But 2x*y:30 (2) Differentiating(1) with respectto r,
tr'rom (2), we get
dA =
Y:30-2x ;f, 4trr*zrr$ + hrh
(3)
Substituting(3) in (L), we obtain
A:30x_2^z , (4) setting = o anasolvingfo, we ger
# $,
Differentiating(a) with respecttb x,
g$=Ro-4x dh 2r- h (3).
d:(
i^ dr

Setting A = 0 Similarly, differentiatipg (2) with respecc to


r,
we obtain
30-4x:O
( n o t e :V is constant)
x= T.Eem Q
' = n r 2 *qr
* z n---
rh I

EXAIIIPLE 2: A closed cylindrical tank (Fig. 9.10) is to be made and solving for dh , we get
with a fixed volume, Find the relative dimensions dr
of the tank , which will require the least amount
-
of material in making it. . dh _-2h (4\
qrr
Solution: This problem arnounts to finding the relation bet-

Fir
ween the height ft and the radius r of the tank of Equating (3) and (4), we have
minimum surface area and fixed volume.
- 2r- h -2h
- --*
lli Let
4
A = total surface area of the tank
: area of the top - rr2
r

H__,
Brc.8.l0
Ab
As
v
=
:
:
prea of the bottom = .rt2
area of the side : 2tth
volume of,'the tank (consmnt)

The quantity to be tninimized is A. Thus


fpom which we obtain the relation h = 2r, The
fesult tells us that the proportion which requires
the least amount of qlaterial in making the tank
with a fixed volume is that the net[nt shoulil
be twice the radias of the base.
A --q+ Ab* A,
ALTERNATIVEsoLuTIoN;'Another sorution
or A =2nr2+?nrh (1) rs to reducethe function to be madea midimum
as a functiqn of a singlevariable.Thus in this

Visit For more Pdf's Books


Pdfbooksforum.com
76 Differential and Integral Calculus -11's'l
rr';rlionsof the Derivative It

problem, we may express A in terms of the


variable r. We start with the two equations (1) l r . , , , , l r u l t eo f a n o p e n box with a squarebaseis 4,000 cm3 .
and (2) given above. That is, b r . l t l r r , r l i m e n s i o n so f the box i.f the material used to make
fr.\ lS a minimunn.
A _ 2 t r r 2+ Z n r h (1)
I t lrl volume of the largest right circular cylinder that can
=: nr2h r ut t'r'o11a circular cone of radius 6 crn and height 9 cm.
V (2)
I t lrc lreight of the right circular cylinder of maximum
From (2), solvefor h. nu' t lrat can be inscribed in a sphere of radius 15 cm.
tlrl rlimensions of the largest rectangle that can be ins-
h -lL I rrr the ellipse 9x2 + 16y' _ 144. The sides of the
Trt' (3)
sk, are parallel to the axes of the ellipse.
Substitute (3) in (1) and simplify to l,rurrr,les trapezoid has a lower base of 16 cm and the
sides are each 8 cm. Find the width of the upper
2V (4) greatest area.
A =- 2rr2 +
r hrl',.zoiclal gutter is to be made from a sheet of tin 22 cm
l')' lrending up the edges.If the base is 14 cm wide,
Differentiating(4) with respectto r, keepingin wrrlt.hacross the top gives the greatest carrying capa-
mind that V is a constant,we get
o{' the bases and altitude of an isoscelestrapezoid is
-dA
--: 2V
+fir --T- l"ind the altitude if the area is to be a maximum.
Qr r'
hrrrlrlrrrg with a rectangular base is to be constructed on a
- -----' !4 , O
Settinr rrr tlrl forrn of a right triangle with legs 18 m and 24 m.
di llrr. lrrrilding has one side along the hypotenuse of the
1,. t'rnd the dimensions of the base of the building fo
4 n r - 4ro = o frfnun floor area.

r. l,rrrgularfield is to be enclosed and divided into four


lrrls by fences parallel to onb of the sides. A total
or 2trr3 : Y (5)
Itl rl(10 rneters of fence are available. Find the area of the
Substitute (2\ in (5). We get ;rt l rr,l rlthat can be enclosed.

:
{r' r,rrr ,rf fl oor area L8 m 2 is divided int o six cubicles of
2tt3 nr2 h
r ll'or &r€zrby erecting two wooden partitions 2 m high
!1,'l l() one wall and another partition 2 m high parallel
h: 2r
rf r'1f 11'pwall. Find the dimensions of the room if the
which agreeswith the result of our first solution I r r r r o r r not f w o o d i s u s e d . '
above.
| \ ,rru.urwindow, consistingof a semicirclesurmounting a
EXERCISE 3.6 ln,r', p,l,',has a given perimeter. Find the radius of the se-
il*, .,, l,' t o admit the mOst light.
A'closed right circular cylindrical tank is to have a capacity
of. l28n ni' . Find the dimensions of the tank that will re- ) t'sttt rrrculor cone of radius R and altitude H is circums-
quire the least amount of material in making it. *rtii..,'i.rlrorrta sphere of radius r. Find the relation between

Visit For more Pdf's Books


Pdfbooksforum.com
78 Dilferential and Intesal \1r1,111's1ions
of the Derivative 79

H and r if the volume of the cone is to be a minimum. Jor,,l t.lrepoint on the curv* y :+ JfS - x2 (first-quad-
t4. Find the lengths of the sides of an isosceles triangle Irnt I wlrere a tangent ttiuy be drawn so that the area of
given perimeter if its area is to be as great as possible. ll,r' trr:urgle formed by the tarrgent lir:e and the coordinate
3le r rs lr minimum.
t5. An oil can with a given volume is made in the shal
cylinder surmount"A Uy a cone. If the radius r of the I r,','t:rngularfieldis to be fenced, one side cf which is the
equal to 314 of its altitude h find the relation bet !l,. ol' a straight river. It is given that the material for the
the rreight I{ of the cylinder for minimum surface F.' ,,p1>ositesides costs P3.00 per meter and the material
can. t, tlrr, side opposite the river costs ?6.00 per meter. If an
,rrrrt,of P600.00 is available,what should the dimensions
16. The sector of a circle of radius r has a given Itr un(:losea maximum arca?
Show that L = 4r for ma:rimum area of the
nrnnufacturer of a certain brand of 'appliance estimates
L 7 , A rectangle is inscribed in the etlipse b'*t, I "'J2 lrl can sell 5,000 units a-yuat at P900.00 each and that
with ehcf, of its sides parallel to an a:ris of the ellil | -ur sell 1,500 units more per year for each P100.00
the gredtest perimeter which the rectangle can have. r.ns(.in price. What price per unit will give the greatest
1 8 . A wire of length J, is cut into two pieces, one of t tts'/
bent into the shape of a circle and the other into { r l,rr,tl rectangular box whose base is twice as long as it
of an equilateral triangle. Find the length of each rrr[' has a volume of 36,000 cmt. The material for the
that the sum of the enclosedareasis a minimum' r o.st,s10 centavos per sq. cm.; that for the sides and
1() Find the length of the lopgest beam that can be m t,nr t'osts 5 centavos per sq. cm. Find the dimensions
rizontally from a conidor of width c into a' rtrll make the cost of making the box a mihknum.
width b if the two corridors are perpendicularto fnfurt{()-grower observes that if 25 mang<r trees are planted
24. A man in a boat 6 km from the nearest point P on a lrr,r't.ilre,the yield is 450 mango_esper tree and that the
shore wishes to reach a point Q down the straight r l.l 1rr,r tr€€ decreases by 10 for each additional tree per
12 km from P. On water, he can travel 4 km/hr and lrrrr, How many trees should be planted per hectare to
5 km/hr. How far from P should he land iir order Irrr l l l t, maxi mum crop?
mize his total travel time? .'R rr'(, to be charted for an excursion. The bus company
2 L . A line is drawn perpendicular to the x-a5is cutting rp,,..fIQ.QQ per ticket if not rnore than 200 passengers
- 2x at 1
rabola y : 4x - xi and {he line y : 12 r rtlr the trip. However, the company agreesto reducethe
and L qespectively. Find the value of x which . ,l' rv€ry ticket F0.05 for each passengerin excess'of
distance from L to P a minimum' ) 1r.r:,S('ngers.
What number of passengers will produce the
and lower vertree-S oI a rectangle he on tl rrrrunlgross i ncome?
22. The upper'5
x' - - v and x2 -- 4y rfspectwely. The siA;s oj t rrl ll.ate
iangle are parallel to the codrdinate il(es. Find the
areaof the rectangle. rr r;1l l that i f y - f(x) , t hen is t he r at e of change of y
' ,, r r. x. H ence i f y : f ( t ) , f i
23. T h e p o i n t s ( 3 , 2 ) a n d ( 1 , 6 ) l i e o n t h e e l l i p s ey 2 + 4 : t hen $i ir t he r at e of change
; r.sfrr,ct to t. If t denotes the
Find a point on the ellipse po that the area of the tirne, then $t iq simply
having thesethree points as verticesis a maximum' | ,,r ;rs the time rate of change of y.LikewiJe,
$1 is the
rr ,'l t'lrir.Heof x. Theserates of changearerelated
24. .Find the point on the curve Y = x3 which is by the
point (4, 0).

Visit For more Pdf's Books


Pdfbooksforum.com
Differential and Integral 'rrrlnc Applications of the Derivative
81
80

E (3.4) dv
--J-- :
dv.dx
+

dt dx dt I lrr, r,olume of the water in ihe tank at time t is


(1)
v=fnr'zh
',,n('('weare to find$f, ttren we have to expressV as a function of
rr lrr F ig. 3.11 and by similar tridngles,we have
rh (2)
result t V {t 3= 8-
IVlany physicgl problems deal with rates of change Solving for r in (2), we get
ties with'respect to time. For instance, when water is po
a tank, t'he water surface is rising with resrrectto tinre. 3h (3)
chan4e in the water level may be expressedin terms of T
Substituiing (3) in (1) and simplifying, we obtain

lVOl'rlme. v= sqT
64
(4)

' I n solving "time rate" oroblems. it is important Differentiating(4) with respectto t -


that all quantities which change with respect to time
dendted by letters. Do not substitute the numerical ual dV dh (5)
voiable until ofter differentiation with respect to the at a-
fuine*.
EXA\{PLE 1: Water is poured into a conical tank 6 m
Substituting 10 andh - 5 in (5),
ff:
top and 8 m deep at the rate of 10 m3 /
225n dh (6)
fast is the water' level rising when the
64 dt
the tank is 5 in deep?
Solving for dh in (6), we obtain
Solution: (SeeFig. 3.11) At time t, let E
dh 12Q m7pln
r : radius of the water surface
dt 45r
h - depth of the water
\\f l'l ,E 2: A ship A is 20 km west of another ship B.It A
\r - volume of the water sails east at 10 km/hr and at the same time
R sails north at 30 km/hr, find the rate of change
h
It is given that ${ : L0 m3 /min of the distance between them at the end of *
l \
l_ required to findff at the instant hr.
5m.
FIG. 3.11
tThis e;ror is commonly committed by thc siudcnl-

Visit For more Pdf's Books


Pdfbooksforum.com
Some Applications of the Derivative 83
Differdntial and Intesal Calculus .

Solution: (See Fig. 8.12) At time t, let --ffi


s = distance between the ships
= lb t/T
x = distance traveled by ship A
y : distance traveled by ship B Substituting the values of x, V, s, d{ ana fi. ln
(2), we get dt dt
wherex : 10t and y : 301.Hence .$f : 10 and#
Cs = - (20 - 5) fr0)J (L5) (30)
T- dt L5 \E

I
v
r0 /Tkm/hr

II Alternatiue Solutiou Another approach is to


expresss in terms of t only. To obtain this, we
substitute x = 10t and y : 30t in (1). Thus
-L
s2 : (20- 1ft)2 + (30t)2

t*
Frc. 3.12
Differentiating -
It is required that we find f,f when t :. + hr.
Using the right triangle in Fig. 3.12, we get the d. - ? (20 , lot) (-- 10) + 2(30t)(30)
relation Ar-
s2:(20--x)'+yt (1) Substituting t : l12, we get
Differentiating (1) with respect
plifying, 10 ruat km/hr
9,|
* v-+
dt " _ct_.
dr S
EXERCISF 3.7

Whent= +,weget I . The radius of a right circular cone is increasing at the rate of
6 cm/sec while its altitude is decreasingat 3 cm/sec. Find the
x.:10(+-): b rate of change of its volume when its radius is 8 cm and itsi
altitude is 20 cm.
y:30(tf=15 '2. A ladder 6 rn long leans against a vertical wall. The lower end
of the ladder is moved away from the wall at the rate of
Solving for s in (f) an<l substittuting these values
2 m/min. Find the rate of change of the area fo_rmedby the
of x and V, we have
wall, the floor and the ladder when the lower end is 4 m from
the wall.
r=ffi

Visit For more Pdf's Books


Pdfbooksforum.com
84 Differential and Integral Calculus t 1,1,lications
of the Derivative 85

3 . A boy 5 ft tall is walking away from a street light at the rate ,;' 'r lrr.h his shadow moves along the wall of the house when
of 3 ftlsec. If the light is 12 ft above the level glound, deter- 1., , , i rneters from the house.
mine (a) the rate at which his shadow is lengthening,(b) the \ lrrrlr post 3 m high is 6 m from a wall. Aman2mtall is
rate at which the tip of his shadow is moving and (c) the rate r rl l i rrrg di rectl y from t he post t owar d t he wall at 2. 5 m ls.
at which his head is receding from the light when he is 24 ll,,rr l'rist is his shadow moving up the wall when he isl.b,m
ft from the point directly below the light l ' , r r rl l r e w a l l ?
4 . Water is running out of a conical tank 3 m acrossthe top and
I t', \'lume of a cube is increasing at thq rate of 6 cmr /min.
4 m deep at the rate of 2 m3 /min. Find the rate at which the
ll, * f irst is the surface area increasing when the length of an
level of water drops when it is 1 m from the top.
} l l ; , ' r s1 2 c m ?
5 . A reservoir.is in the form of h frustrurn of a cone with upper
,*.rrrrlrs poured at the rate of 10 m3 Tmin so as to form a co-'
base of. raclius 9 ft and lower base of radius 4 ft and altitude
frr' rl rrrl* whose altitude is always equal to the radius of its
of 10 ft. The water in the reservoir is x ft deep. If the level of
l.ro, l"rnd the rate at which the area of its base is increasing
the water is increasingat 4 ft/min, how fast is the volume of
l l r ' 1 ,t l r c r a d i u s i s 5 m .
the water in the reservoir increasingwhen its depth is 2 ft?
NOTE: The volume of a frustrum of a cone o{ upper base A t','rgh whose cross section is an equilateral triangle is 6 m
radius., R and iower base radius r and height h is [,,.p ;rr(l 2 m wide across the top. If water is entering the
V- $zrh(Rz+f +Rr). lr'rrrrilr :rt, 15 m3 1min, at what rate is t*re water level rising in
.J
6 . At noon, ship A is sailing due east at the rate of 20 kmihr. ll',' I'orrgh when it is three-fourths full?
At the same time, another ship B, 100 km east of ship A, is l,rf ,'r is poured into. an inverted conical cistern of altitude
sailing on a course 600 north of west at the rate of 10 km/hr. l,' lt ;rnd radius of base 5 ft. If the water.level rises at 86
-
tlow fast is the distance between them changing at the errd Jl find the rate at which the dry surface of the inside of
",rr,
of one hr? When witl the distance between them be least? ftr , r,rrr is decreasingi.whenthe water is 4 ft deep.
t. A ship is sailing north at 2J km/hr. A second ship sailmg A ,1' l r.ri cal i ron ball 8 cm in diam et er is coat ed wit h alayer
east at 16 km/hr crossesthe path of, the first ship 85 km ul :,,, ()l ' uni f' orm t hickness. I f t he ice r nelt s at t he unif or m
ahead of it. How fast is the distance between them changing trr, ,,1' l0 cm3/rnin, how fast is the thickness decreasing at
one hour later? When are they closest together? l l ' , r r r s t a n tw h e n i t i s 2 c m t h i c k ?
8 . Two roads intersect at 600. A car 10 miles from the junction
'moves towards it at 30 mi/hr while a bus 10 miles from the fl ,rl ,' r' { ' l oW s out of a hem ispher ical t a'nk at a r at e which is4
l.,n,'\ the square root of its depth. If the radius of the tank is
junction moves a\ryayfrom it at 60 mi/hr. Calculate the rate
t tt lrow fast is the water level falling when the water is 4 ft
|: at which the distance between the vehicles is changing at nl,r 1r"' llint: Use the formula for the volume of a spherical
the end of.2O min. - h)
rt l r r ' n r l r r Li ,. e . \ ' = 1 " h'(3r
9 . A bridge is L0 m above a railroad track and at right anglesto 3
it. A train running at the rate of 20 m7s passesunder the lr. t rlnloar Motion
center of the bridge at the same instant that a car running
15 m/s reaches that point. How rapidly are they separating \ r,,rl! which lnoves in a straight line is said to be moving
, 3 secondslater? fi' 'r,'tu' tnotion. If the moving body is small in comparison
10. A light at eye level stands 7 meters from a house and 5 me- frt,.,,tr',lrrnt:€ it covers, then it is customarily referred to as a
\
ters from.a path leading from the house to the street. A man
t walks along the path at 2 meters Der second. Find the rate l, L lx,the directed distance of a particle P from a fixed
r I , r r i r coordi nate line ( Fig. 3. 13) . I f t he m ot ion of P along

Visit For more Pdf's Books


Pdfbooksforum.com
86 Differential and Integral
Ag'pli<:ationof the Derivative 87

the line is given by the equation s = f(t), then the veloci


the acceleration d are defined as follows:

As: ds
,r: $: B t 2- Izt+ 9: B(t - t) (t - B)
E (3.5) lim
At*o at dt
dv
a:aT: 6t-al':6(t-2)
E (3.6) lirn A - du
A;;" at dt Hencev: O when t : 1 and t _ B

and a-Owhent:2

We obsenre that

(a) When t ( J, v ) 0 and a ( 0. Hence during


the time L < 1, the particle is moving to the
. l.IG. 3.13
right with decreasingvelocity.
Thus, we note that the uelocity is the time rate of (b) When 1 < t < 2, v 1 0 and a ( O._Hence
the distance while the acceleration is the time rate of during this time interval, the particle is
the uelocfty. The absolute value of the velocity is called moving to ihe left with decreasing velocity.
of the particle. (c) When 2 < b < 3, v ( 0 and a ) 0. Hence
during this time intenral, the particle is mo- -
The sign of the uelocity determines the direction of ving to the left with increasing velocity.
of a particle P relative to its starting point. It can be sh
(d) Finally, when t > 3, v ) 0 and a > 0. Hence
(a) If v ) O, the particle P is moving to the right. during the iime interval t ) 3, the partictb
is moving to the right with increasing veloci-
(b) If v ( O, the particle P is moving to the left. ty.
The sfin of the acceleration determines whether the
increaseswith the time. It can also be shown that The motion decribed above is shown schema-
tically in Fig. 3.14. Note that when t : 0, the par-
(a) If a ) O, the velocity v is increasing.
ticle is at s _ 3 and moving to the right with
I v : 9. The particle continues to move to the.right
(b) If a ( O, the velocity v is decreasing. until t : 1 when v : 0. Since s : Z when t : 1,
I
then the parficle momentarily stops after moving
EXAMPLE 1: The motion of a particle moving on a 4 units to the right of its original position when
ll
line is given by t : 0. It then 1s{€rs€sdirection and moves to the
left until t : 3. When t : 3, v : 0 and s : 3.
'r 1s + 9t+ 3
il
Describe and diagram the rectilinear Thus it comes to stop again upon reaching its
tl r> o. original position. Then it turns right and moves
off to infinity,.
al

Visit For more Pdf's Books


Pdfbooksforum.com
88 Some Applicatioir of the Derivative 89
Differential and Integral

At the highest point, v : O. Hence from (2)

O=96-32t
t :3sec.

F I G , 3 . 14 This is the time required to reach the highest'


point. Substituting t : 3 in (1), we get

One of the most important types of rectilinear s: 144 ft.


that with constaftt qcceleration_ For example, a freely falli
near the earth's $urface moves wittr a constant accelera Hence the maximum height attained by the body
freely falling we mean that air resistanceis neglected.T is 144 fr.
tant of acceleration due to gravity is denoted by g and is
cally equal to 32 ft/secz or 980 cm/seC?. EXERCISE 3.8

Conslder a body in rectilinear motion which moves In each of the following, s (in ft) is the directed distance of a
upward or downward. The effect of gravity is to srorv rnoving body or particle from the origin at time t (in sec) on'a
down if it is rising and speed it up if it is falling. Supposea ,'oordinateline. Describeanddiagram the motion foryt 2 O.
thrown vertically upward from a point A with an initial
vo. It can be shown that its distance s ft from the starting L s:2tt -75t2 +36t
-
A at the end of f sec is 2. s=t3-912 +24t+3
: t . s = t 3 - - L 2 1 , 2+ 5
E (3.7) i=vot*16t2 (vo in .t. s-tr-9t2+15t+4
-(vo
E (3.8) s : vot -- 4g\t2 in l'rnd the valuesof t for which the velocity is increasing.
Note that s > 0 if the body is above A and s { 0 i f i t i s
S: trr-6t2+4
If the body were thrown downward, then we consider vo
5- t3-L2t2+5
EXAMPLE 2: A body is thrown vertically upward t4 - 8t3 + 5
glound with an initial velocity of g6 ftlsec.
(t - 3)o
the marimum height attained by the body.
I f s : V8t + i/ 4t, find the velocity and acceleration when
+- 2.
Solution: Substitutingvo = 96 in E (3.7), we have U. -

If s : t3 - t2 , find the velocity when the accelerationis 2.


s:96t- 16t? (1) If s = 3t2 - L6t-2, when will the accelerationbe zero?
An object is thrown vertically upward from a point on the
Then by E (3:5),
ground with an initial velocity of 128 ftlsec. Find (a) its
velocity at the end of 3 sec., (b) the time required to reach
v=96-32t
the highest poirri, and (c) the maximum height attained.

Visit For more Pdf's Books


Pdfbooksforum.com
90 Differential and Integral Calculus chaptep

13. A body is thrown. vertically upward from h point on the


glound. If it attains a maximum height of 400 metets, find Differentiation of
its initial velocity. Transcendental
L4. From the top of a building 42 meters high, a body is thrown Functions
vertically upward with an initial velocity of 36 meters per
second. Find (a) its greatest distance from the ground and We shall discuss in this'chapter the differentiation
of a new
(b) its velocity when it strikes the ground. r:la.ssof functions. These furictisns which
are not algebraic are
t:alled transcendental funettons. The trigonornetlic
1 5 . An object thrown vertically upward from the ground rea- funeti";r- ;;
their inverses, together lvith the logaritrrmic and
ches a certain height after 2 sec and retums to the same exponential func-
tions, are the sirrlplest transcendental functions
height on descent, 8 sec later. Find its initiat velocity and the
height in question. 4.1 The Function

consider the function f defined by the equation

f (u) = ai$!L
't'his
functibn assumesthe meaningless form,${or
u : 0. Howevlr,
the limit of this function exists roheo-,rappiJacnes
zero. To prove -
this, consider Fig. 4.1 where arc AC subt€nds
; ;gt" i-"rr"""a
rrr radians) at the center 0 of a cirele of radius "
r.

FIG. 4,1

91

Visit For more Pdf's Books


Pdfbooksforum.com
g2 Differential and Intesal Calculus nt iation of TlanscendentalFunctions 93

lt'lurion:tim =+* = .fim(f -ry)


be the perpendicular to OA and BC be the tangent
x+o
to the circle at C. From the figure, we note that
lim 1+lim sTx.
Area of AODC ( Area of sector AOC ( Area of AOBC
x+o x+o
or + (oD)(DC)< + (oA)'(u)<* (oc)(cB) (1)
1+1
By Trigonometry and wrth r : OC, we get the following relations:
2
OD = rcosu
DC = rsinu . l ; . 2 . lim sin2 3lcosx
'\
x+o
CB = rtaRu = " sinu
t r -

cosu ]rluti<lrt:

Substituting these values in (l)-we have


6* sin2 3Tcosx
x'
fi* sin'_3+Sgtx_ 2
xz9
x90
f 12 cosu sinu ( * ,, ,, ( *rr sing
cosu rim(*-s??xecosx)
x+0
Dividing each term of (2) by *r' sinu, we get
u*#ti*Yhmecosx
u21 x+0 x+o x+o
cosu<ffi a;m ,
1.1.9.1
Taking the reciproeals of the terms in (3)
g
I - sinu-
(4)
cosu u
Frorr. (4), we note that 4s u+ 0, cosu+landoafo+l. Since ttf u
lies between cosu and both of whicn approach one as u
approaches zero, then +g ".tr',' 1. We now formally state this fact nrch of the following limits:
as a theorem.
l# lim cosxtgn2x
T (4.1) If the angle u is in radians, then the ratio S|.upproa-
l?.,, 6.
x+o SmZx
chesunity as u approcheszeto.In symbol, tan4x
hr
-'fx
7. lim +-
L10 Ilm -slnu
: I
I r rt x+o Jx
u
u.o I - cosx
lit
?Fr- 8. lirn 5 -,cgsx
* l=fill I r tr x+o 4x2
EXAMPLE 1: Evaluate lim
x+o x

Visit For more Pdf's Books


Pdfbooksforum.com
Differential and Integal lrl.ron of Transcerldental Functions 95
94

4. tim 4xz :x 9. lim 1- gos4x ,ly- .-


lim a.L
x+o 1-cos2 X+O xsin4x rlu . ^ A u
Au (J

'r@
10. lim 1- coszt = lim
x2 + 4x x2 cosx Au*o
Au
5 . lim X+O
x +o sin2x
Zcos1u + # a u ) _ s i *na u - (why? )
lim
Au, o Au
4.2 Differentiation of Tligonometric Functions cos(u + j-Au) sin * lu
- Iim
The following formulas are uted for,differentiatihg Au*o t6u
metric functiont. Tt e symbol u denotbs an arbitrary diffa
function of x. = lim cos(u +*au) lim en i a"
Au,o Au'o+Au
D12:
$sinu): "ort* -_cos(u+0).1

:-sinu$|
f t"or,r)
Dt3: rfore,
dv
+ -- cosu
ou -
. D14: sec2"*
St*rrrr): trptying both sidesot we get
*
D15: t) :-csc2
$t"ot "* -ry du-
- cosudu
-
du- dx cx
D16: : secutanu
t S{r""rr) S .dy- du
COSUi-
&- .ox I

ib
D17: : -cscuc*"
$t"r.o) * r. y -- sirtu; then I
I

C
I

we shall give the proofs of the first three formulas. Tlt =


-
U$t*ittut cost,#
of the remaining three should be carried thropgh by th9
In proving D12, we shall use Definition 2.1 (Chapter 2) l)t 8:
b proof.
lr?r)ve D13, we may use agdn Definition 2.1 and L10 but

r( Proof of D12:
Let y = sinu where u is a function of x; Then we
, its pr.oof !s based on the result already achieved for
t r t . b y usingD12.

rrl sin (u + Au) - sinu.By Definition 2.1,

Visit For more Pdf's Books


Pdfbooksforum.com
96 Differential and Integal t trtferentiation of Transcendental Functions 97

In trigonometry, we have the following relations: Therefore, we have

sinu = cos ( + zr-- u) (1) { 1 t a n u ; = s e c 2U "=4- 9


& dx
.orr, = sin ( + z_u) (2) The following examples illustrate the use of the formula for
rI rf fcrentiating trigonometric functions :
Differentiating (2) with respectto x,
dd.rr I:XAMPLE 1:
fr(.o*u)=#sin( f n-u)
Find ir v = sin4x
{}
= c o s(
*r-r)*(|r,-uy Solution: We note that V : sin4x takes the form. y = sinu
= sinu (- with u = 4x. Hence
**)
Therefore, dy = A
'by i
cos4x# (4x) Dtz
Tx ox
d
(cosu)--sinu$ t-

a; : cos4x (4)
Proof of D14: t

We shall.useD12 and D13 to prove D14. Since , I

dv .^ ;
tanu = sing IXAMPLE Find#ifY=sin34x
. cosu

Then differentiating both sides Solution: If we write y = sin3 4x as y: (sin4x)3 , then it


takes the form Y = uD with u - s i n 4 x a n d n - 3 .
rl $r,*,,)=*[H) This suggeststhe use of D7. Thus

-{I- = Bsin?a* * (sin4x) byDT :


- - .d qx qx
cosu
ft-tri""l SIIIU r
ox
(

- = Bsin2 4xcos4" (4x) by D12


- *
sinu ( - sinu)
: 3sin24x cos4x (4)
- cos2u
= 12sin24xcos4x
--
cos2u* sin2u du
|}--- cos'u dx As we get more familiar with the formulas and their uses, we
flnv Jrerform some steps mentally and thus shorten our solution.
{ 1du
-- l'r,r instance, in Example 2, we may omit some lteps given above.
cos- u (Ix l'lrrrsin practice, the problem is worked out siitipty this way:
f,

Visit For more Pdf's Books


Pdfbooksforum.com
98 Differentialand Int€gal Calculus I )i fferentiation of Tlanscendental Functions
99

v- sin34x
d{= The vdlume of the cylinder is
3sin2'4xcos4x(4)
dx , V:nrzh (1)
, - 12sin24xcos4x Since we are going to solve this by use of trigo-
nometric functions, we introdgce O as our new
tt y: tana5x variable. From the right triangle in Fig. 4..2, '
EXAMPLE 3: Find
# *"
obtain the following .et tionr: . :
=
Solution: v: tana 5x .h 3Ocosg (21
r = 15sing (3)
dv
--l* =
4tan3 5xsec25x (5)
dx Sub$tituting (Z) and (A) in (f) and simpliyi4g,
Z}tan3 5xsec25x
V = 6T5Oz sin2 Ocos{t (4)
f-

EXAMPLE 4: Find the height of a right circular cylinder of ma'


Diffqrentiating (4) with respect to
ximum volume that- can be inscribed in a sphere Q
of radius 15 cm. rlv
= 6750r f sin'O(- sin0)+ cos0(2sin#cor+)
Solution: This can be solved by the method used in'Chap! # I J
^J
ter 3. In faet this is Problem 4 in Exercise 3.6. .'
= 6750r (sin0) (2cos2g- sin24) ..
Tlris time, we shall solve it by using trigonometrie
functions. In Fig. 4.2, we have
Setting = o,
h = height of the cylinder #
r : radius of the base
Let V : volume of the cylinder. We are asked to 6?50n (sin$) (2cos24_sin2 O) =
0
e find h for maximum V. a

Then we have
rft

sinO = 0 (discard this value)


-
-) and Zcosz0- sin2f = 0
2cos2f- (1"- cos201= g

(b
T-
h/z
3cos2o_-1=A
./T
c o s t t =Il T = : t

Substituting this value in (2), we obtain


FIG. 4.2
h = 10.,uF cm.
JI

Visit For more Pdf's Books


Pdfbooksforum.com
100 Differentid and Int€grd Calculur tt r,rl,ionof Transcendental Functions 101

l}r\ =y
EXERCISE4.2 \, x

Find *O simplify the result wheneverpossible }|lt)' I ycosx = 0


#
lltr' following problems by making use of Trigonometric
1
1. t=**-+sin2x
1 I t,he dimensions of the right circular cylinder of maxi-
2. y=sinbx-fsin'5x nr lnteral surface area which can be inscribed in a sphere
tulrus 4 in.

3. v=sin24x+j-"ora* rt rcngth of a rectangular bean is proportional to the


Ith and the square of the depth. Find the dimensions
llrr. st,rongest beam that can be cut from a circular log of
4. v=3xco.+-9sinf trr R .
3 3 ,_- 1 ^__3_-_:_--
b. V = * -g.slnxcosx * cos"xsrnx thc length of the shortest ladder which will readh from
T-X T ground level to a high vertical wall if it must clear an 8-ft
6. V =x2sinx+ 2xcosx- 2sinx tcrrlfence which is 27 ft from the wall.
tlrr. volume of the largest conical tent that can be cons-
7. V:sin(x+4)cos(x-4) 'trrf with a slant height of.12 ft.
-
^ 1 - cos4x tlrc area of the'largest regular cross that can be ins-
8. y=ff I in a circle of radius R. (A tegular cross is a square
nunted by four equal rectangles.)
9. y:3tan2x4tan32x
l*hk,r 10 ft long leans against a vertical wall. The upper
10. Y : sec24x + tan2 4x -i rlrps down the wall at 5 ft/sec. How fast is the ladder
x when it takes an angle of 30o with the ground?
q 1-
ll y.3sex-fcsc'x h ,l' the equal sides of an isosceles triangle has cons-
l'rrgth of 4 ft. If the angle f, between these sides in-
- 12 .y, secax - 2tan2 x r lt the rate of 10 rad/sec, find the rate at which the
rnur('reasingwhen O=
C 13,. Y: secr2x- 3sec2x t
rr1';rol,enuseof a right triangle is 25 ft. If one of the acute
-
L4. Y : csc4x- 2cot2x f'r urcreasesat the rate of. 4 degrees'per second, how
rr llrt' area increafing when the angle is 30 degrees?
e
brt 1 5 .y =- f c o t ' + . c o t 3 f - scot+- *
r.rr I ii,tion of Inverse T?igonometric Functioru
16. cos(xY)=x-Y
( lr "', ;rll from trigonometry that
L7. sin(x+y):x+.y
rrn r\rcsinx iff x = siny and- + S v S 1l
2

18. xcosy: sin (x + Y)


fl

Visit For more Pdf's Books


Pdfbooksforum.com
102 Differential and In Iron of Transcendental Functions. r03

Note that without restriqting the values of y l n rl 1 du


=
I-i,i j r ; h ee q u a t i o ny . A r c s i n x ' d o e sn o t dbfine a
rlx
( Arcsinu)
Vr:;r a;
,1. du
(Arccosu)=
The reason for ihis is that for any value of x in the in rlr J# d*
there are infinitely many values ol y which will satisfy rl du
tion y
(Arctanu)=
Arcsinx*. However, with this restriction, rlr 1+;r- dx
for each value of x in [- 1, 1], there is a urriquevalue rl du
instance, (Arccotu) = -r-.
rlr ;-- dx
y=Arcsin(*l=*
rl =
(Arcsecu) du
rlx *ft- dx

y = Arcsin( - 1) = - + (Arccscu) =
du
u.1f[z_f a;
The notation Sin-txis often usedfor Arcsinxbut in
we shall use the "Arc" notation.* The definition of the fltx:
inversetrigonometricfunctionsareas follow: l .t' t Y = A fcsinu (1)
Y = Arccosxiff x = cosyand 0 7 y 1n l'lr,,rr u : siny (2)
-
V = Arctanx iff x = tany and - *< v <+ llrf fcrentiating (2) with respect to x
clu dy
C O S- V- ; - (3)
V = Arccotx iff x = cotV and O ( y ( rr clx rlx

y = Arcsecxiff x = secyand - n S y < - (


.+for x l{olving(a) forS

- o(y<+ forxZ I dyldu


&-: T; (4)
lr y = Arccscxiff x = cscy ancl- n ( y ( - f f o r x S - ""rll
f l r r t ( : o s y= V 1 s i n z y = V 1 - n z T h e p o s i t i v es i g n
.rf llre radical is chosensincecosv ) 0 for
- o(yS+ for x,] I .- lt
, . )- y \ t ) . Flence(4)becomes
-
The following formulas are used for differentiatinj
trigonometric functions. The syrnbol u denotes an arbi 4Y-= I -_ du
hr ferentiable function of x. dx Tltr]-- dx

t i u l r s t i t u t i n g( 1 ) i n ( 5 )
The studentwho is not so familiar with the propertiesof unversetr
\h functionsshouldrefeqto any standardtext on trig-nometryfor revibw.
q ** The notationSin -rx is consideredinconvenient
by somepeoplesisceil
readas"sinxwith exponent-1" The- 1 i1 thisexpression
is not an
il fore Sin-rxdoesnot mean(sirx)-t or
sffi^

Visit For more Pdf's Books


Pdfbooksforum.com
of Tbanseendental Functiorts 105
104 Diff erential and Inteea'al Ilifferentiation

Proof of.D22: 4JL=


$ ts*l
-_1 -

Lqt y = Atcsecu
dx
Vl -(gfl
Thirn u - secy = I .,
Differentiating (2) with respect to x vl-:Til-
g = 'ecYtanY I:XAMPIE 2. If y - Arctan
i, ti"A'ff
0x ff
Solution: By D20, we have
tor $f
solvins(s)

d' + v= 1 d u
-
d y =--
dx
+
1 J.
1---
/"\'
*F)
c)
il- secYtanY dx \T/
'l/se =T= {v' - =L,
But secy: u and tany :
positivesign of the radical-is chosen since tany 1+#
:"; 1;'? - + ."0 o s v' < F). substituti
2--- z'
4
valuesin (4).
16+x2
qL=
- +s-
dt uvlf- 1_- dx }:XAMPLE3. A ladder 25 ft long leans against a vertical wall.
If the lower end is pulled away at the rate of
Since I = Arcsecu' we finallY get 6 ft/sec, how fast is the angle birhreen the ladCer
and the floor changing when the lower end is
e $tor..ecu)-;#* 7 ft from the wall?

- the re Solution: In Fig. 4.3, we let x = distance of the lower end


The studentis urged to give the proofs of of the ladder AB from the wall CB and let &=
illustrate the use'of
- mulas. Here are some examples to angle between the tadder and the floor CA. TlIe
las above. ., t.d+ lx
r-r wantto find wnenfi = 6 ftlsecandx:.?.
ff
EXAMPLE 1: Fincl# tt V = ArcsinSx Since cos0 =
*,
-
.- = Arccos p
-- +
Solution: Since V : Arcsin3x takes the form V ihen
25
L .- whereu : 3x, then we use D18' Thus
-l

d Visit For more Pdf's Books


Pdfbooksforum.com
?
106 Differential and Integal ll I f r.rr.n tiation of Tlanscendental Functions 107

Differentiating with respectb t, x


I r Arcct"
qf= -1 1 T
9
dt E. z e dr 1 4sin*
625 I v iArctan Tffi
' Arcco, * '''tt4:7
Sqbstitutingx = ? and = 6, we get 2
$i
l y x tr7=@ + fArcsin2x
d + 1=- tr
rao/sec. y . t/ x2 -'4 - 2Arcsec
dt 4 fr !
The minus sign indicates that O is decreasing. -
y Arccotx + Arctan
#

y = Arctanx + ArcsecV-ffi

y = xArcsin2x - 2x + 2 t/fr Arcsinx

y = (x - 1) 'Jffi - Arccos(x-1)
YV
Arcsini+Arccos?=t -
x.
ArctanT= x - V

u - v*'Il + aArccot(lG)
\f
-,
fr y -t/T-T +aArcsih*
-
EXERCISE4.3
.3, Find 9-.rra sirnplify the resirlt wheneverpossible f o t' = ab Arctan
* ^ \ /ry"\
b
ox )
G
1. V=Arqsiny'1-x'? ]r y = Arccotilh
^x
I flr 2 . V = Arccos
[! t,lr' the following problems by making dse of irrverre trigono.
rrrrtrl(' functions.
4
./ 3. y: Arctan;-
II I'he lower edge of a picture is 4 ft, the upper edge g ft
!-,
irbove the eye of an observer. At what horizontal dlstance
olt 4. V = ATCCOI'ton2' ) should he stand if the angle subtended by the pictu.e ir-"
1 rna:cimum?
5 . Y = ArcsecV 4x+
.)l

Visit For more Pdf's Books


Pdfbooksforum.com
108 Differential and Integral Calculus r, n I ri rtron of ' l ' ran.scendent alF unct ions 109

rrtrrrrl)(]re is a nonterminating and nonrepeating decimal and


22. At what point on the line x = 4 does the line segment from u. r'afl be obtained to any desired accuracy. [n practice, the
(0, 0) to (0, 6) subtend the greatestangle? r urr;rtcvalue assignedto e is 2.718.**
23. A sedrchiight, 1-l2 mi from a straight shore, rotates at the l'1r,.graprhof y - (1 + x)r is shown in Fig. 4.4. It shows
rate of 2 rev/min. How fast is the spot of light from the f rrr':rtly that as x approaches zero from the left, y decrea-
searghlight moving along the shore when it is 1 mi from the I lpJrroaches e as a limit. On the other hand, as x approa-
point on the shore nearest the searchlight?
roru from the right, y increases and likewise approaches e
24. An isosceles triangle has legs 10 cnr. The base decreasesat flmtt. Furthermore, we note that asi x becomes positively
the rate af 4 cmlsec. Find the rate of ehange of the angle , ), approaches L as a limit and as x approaches- 1 from
at the apex when the base is 16 cm. t, y increaseswithout bound. Hence V = I and x = - 1 are
26' e, hdder 14 ft lorrg is leaning against high with the rtr.sof the curve.
" ryl"-e 8it
upper end proje"iittg ouerlhi fence. If the lower end s'lides
In leneral, if u iB a funetion of x, we define e as
from ihsfence at the rate 9f 2 ftl sec, fina lfte rate at
"iri.y the angle between the ladder and the gro3nd is chang:
which Ll1: e ==Iim (l +_u)l/u
inq when the upper end is iust at the top of the fence. U-. O
26, A searchlight is trained on an object falling under the in'
fluence of gravity from a height of 500 ft. Find the' rate x
at which th.e beam of light is following the object when the
object is 100 ft from the grodnd. Assume that the search-
light is 200 ft from the point where the object hits the
gFound'
1 {

4.4 Thetunctions(1 + ..)+


The function defined by the equation
.t

rg I
y = (1 + *)* I
I
-D
assumes tfre meaninglessform 1* for x = 0. However, it can be
'the
I
I
rBi shown that limit of this function exists when x approachec {-
I
zero.* This li-i! is denot€d by e. That is, I lo
lbt I
t------T
lim
XTO
(1+x)* =.
I
I
I
I
l*
I
hr I
2
\,,
fA rigorous proof
showing the existenceof this iirit i, beyond the scopeof this
rl book. vuluecan be computed by'the infinite sericsmethod which is not discuesed

-rl

Visit For more Pdf's Books


Pdfbooksforum.com
114 Differential and Integral Cdcultts' t trfferentiationof rransgendentalFunctions l1b

Since y - logbu, t[en we have 41n (2x + 1)


d 1 - .du
dy_,
Oi(los6.r)=iGogse)ff =*'ifftzt
7
dx
DzScanbeobtaineddirectlyfromDz[.Notethatifwe
replace b bY e in D24, we get =8
= (log"e1 2x+L
ff Cos".'l# S
I:XAMPLE 3: If y = 1n l=.find *.
rx_ +. dx
: 1n e = 1 by P4' The equation abovc
But log.u = 1nu and log"e
finatly Secomes D25, that is solution: This can be solved by direct application of D25.
But we shall solve this by applying first R2, pB;
9(r"t')=+*
clx and P2 before using DZlo. Thus

EXAMPLE 1. Find Y = logo (4x + 3)'


*tt

Solution: Let u : 4x + 3. Then bY D24' by R2.


gI = -Lt (log;e)(4) {
dx 4x+
by P3
_ + (logrg)
4x* 3 I I
=+ | rn 1x+ 4) - 1n.(x-4)| UyeZ
LJ
q EXAI\{PLE 2: Find# tt V = 1n (2x + l)a d y r [1 1l .
-- =;-
dx ' r--T
x+4 J
lx+4
-r
Solution: Let r, : (2x + 1)o . Then bY D25, -4
= F - -

Gii 1 x2-16
5lY=. 4ex+1)t2:
TEh
dx (2x + 1)'
EXERCISE4.4
rl rr i
rq 2x+L l ,rrrl )^,{ and simplify whenever possible.
clx

t?n
Alternative Solution: Another solution y =log\ffi-
I
\rr1, apply first P3 and then use D25. Thus
Elll I Y = log sin2 4x
Y= 1n(2x+ 1)o
dlrl
Visit For more Pdf's Books
Pdfbooksforum.com
116 Lifferentid and Intbgnl .rr r''n of TranscendentalFunctions 117

' ,\'t)+2Arctan|=O
3. v=losd @
f'r
4 . y= r " el ( x -1 ) ' ( x* z l oI tt ) xy
LJ
5. V= 1n(x+ 3)a r y l r r x= L

6. v = trn(* i .,/Fr Il rnic Differentiation

7. v= ln'(x+ 3) rr.etion, w€ shall learn how to find the derivative


rr which is expressed as a product, quotient, power
r. v=lnJ#* lwo or more differentiable frrnctions of x by a pro-
'wn as logarithmie d,ifferentiotiwt., Thls procedure
ti".\'
0. v] -'r
= ln /f- tfrr.following steps:
FEin. I trlc Lhe natural logarithm of both'sides of the equa.
S'n which defines the function.
10. V'=xArctanx- ln ,lCrT
|nrplrfy the right mernber of the r€sulting equation by
11. V=ln(secx+tanx) Drlrng use of the properties or laws of logarithms.
tltlf ,.r'cntiatewith respect to x and,solve fo. ff.
L 2 .r = f n $

x2(r+1)
If y - (2x + r) /3?F-$-; tino ff by logarith-
v = ln mic differentiation.
(x+ 2)r
B 'l'aking
'''@rl the logarithm of botJr sides,
y = xArcw2x- + 1n (2x +
-
lnv: ln (2x + 1)\rE-i:5
1;
Ei
v = xa(1- 1nx')
lrr(2x + 1) +-f ln (3x + 5) b y p l , R Z ,p g
-; y = *r vCffi + fa' ln (x +',/ffit )
fe rr.rrtrat,i ng
w i th res pect t o x,
y = x A r c t a n +- ? 1 n ( a 2 + r 2 ) 'ii
q
,*-J, e)+f $; tsr bvD25
)) 18. V=1n(lnsecx)
fif1 ^'|ri
Iltt l'-*l " + = 3 I
19. V = 1n(1n4x) * llx+l 2 ( 3 x + '5J ) l ,
E[l L

20. siny - ln(r+Y) = W3,.r+ l)-t s(2*+1ll


j|lll

Visit For more Pdf's Books


Pdfbooksforum.com
722 Differbntial and Integral C l trf ferentiation of Transcendental Funetions 123

Then EXAMPLE 4.6

lny = ulna Trrr<l9{ *O simplify whenever possible.


d,x
Differentiating with respect to x
I y- 34x
1dv -
ya;= r n ad6u;
L+ 2x
2 r -
! t - -

L- zx
dv ,< ,du
ffi: Y (rnarai;- il y - 4xln4x

Replacing y by au, we obtain D27, that is a y = s-4x


rl

$t""t = aulrna)$|
0 y: exelH
oZx- r
6 Y:lnffi
Note: D28 may be obtained directly frc
by replacing a by e. ? v - ln ("**2)
t ' : i l

EXAI\{PI,E1. If y- 42x,find$f: i y: *2x

tf y=x€x
Solution: Y-42x
ftl 3x+Bv-6
g = +2x1tn+)
(z)
<' cx ll *Y+zv-8
,- 42x(2ha)
iID fr ,-'xv+h(xy):3
= e2x 1tnt6;
G qr lr eX+eY-exeY
EXAMPLE 2. If y- usinx ;find
1-J' dx
lr .Ysirx:
*
Solution: y - gsinx
-
lr, ,,ln4x1"In4y-t
^- : (cosx)
**
t7
I
".-)
"sinx lfi ,,** v:1tt+
*t
l't *Y+ eX: a
4
Visit For more Pdf's Books
Pdfbooksforum.com
L24 Differential and Inteeral Calculus Gr'.ntiationof TranscendentalFunctions
726

18. Find the value of A so that Y : Ae2t will satisfy the equation H2,
y"- 2y'- 3y- e2t. fanh2x* seeh,x:1

19. Find the minimqm value of y : 4e^ * ge-x. H3. coth2x- csch2x=1

20. Find the ma:<imum point of the graph of y : "-*t H4. sinh2x = 2sinhxcoshx

2L. Find the area of the largest triangle cut from the first quad- H5. cosh2x = cosh2x * sinh2x
rant by a line tangent to V : e-2x
= 1* 2sinhzx
:2cosh2x- 1
4.9 The Hyperbolic Functions
Certain combinations of the exponential function ex and l'l,E 1. Prove that cosh2x - sinh2x : 1.
e-x occur frequently in mathematics, science and engineering.
These functions are called 'hyperbolic functions*. They are de' ?rrrrtf: Since by definitions 4.1 and,4.2
fined as follows:
Fnhx)- u*;"-* andcoshx: .1!#-
DEFINITION 4.1 sinhx = e1=-ql*
2
tl rr,n, w e have
DEFINITION 4.2 coshx = ex + e-x
-
sinhx * n h zx - s i n h ''-x = \hT* +- ) -t -\ -T\ t)- f u \ t
DEFINITION 4.3 f,annx = ----;--
cosnx

DEFINITTON 4.4
co.shx
cotnx : JifrE,x etr+ Zgxe -x 4 e-zx e2x-2exe
-x
+ e-2x
4
Gi
DEFINITION 4.5 sechx :
t.D,
e2x+ 2* e-?x- e2x+ Z- e-2x

G DEFINITION 4.6 cschx: # 2 + 2


The notation sinhx is read "hyperbolie sine of x". The 4
others are read in the sarne manner. =l
The following identities can be deduced directly from the
-t
definitions of the hyperbolid functions. t . E2 . Prove that sinh(-x) = - sinhx
.al
Hl. cosh2x- sinh2x= J
}l ' r,f : By Definition 4.1
lL
-t

fThey are called hyperholic functions becausethey can be related to a hyperbolr. x) :- e (-x)
-i sinh (-x) = "(-
Recall that the trigonometricfunctions are also calledcircular functions becauseof thob
relation to a circle.
3t
Visit For more Pdf's Books
Pdfbooksforum.com
132 Differential and Iritepfal rorrof Transcendental
Functions 133

Finally substituting (1) and (6) in (4), we get the desiredf


: 1 du EXSRCTSE
4.9
$tstrri-'.r)= y'uz*1dx
r' undsimplify wheneverpossible.
Alternative proof of D35:
t-
Let u be a differentiable function of x. Then by ^/x, - 1
llrh
tion 4.8n
a r . t ,1 f
sinh-ru= ln(u+l.fiFTTl
bnh 1 (t-z*)
Differentiating with respect to x I
Fvlr 1x+ 1;
/
_1
.d du
5ir.i"tt-lu): +(t* -Jr- \ rotlr 1 (coshx)
u+rffi\ ffiT/ dx
t(#J
r,rrrr,
I
__
u+fi?E )tr r.xf1 1 lsec&r1
-
-1du r.,'lr t ltarrxy
dx
"ml 1,,1u,?Tl* * sinh-1x
EXAMPLE 1. If Y : sinh-l 4*, finaff' ")
l r , 1 1 - - g x 2) + 2 t a n h - 1 3 x
-il
Solution: SL-
\rn dx v(a;F+l $r+*t
-i31 4
Qlnr y'16x2 + 1

EXAMPLE2. I f v :
-1(z*-
cosh 1),findff'
thrrr
Solution: dy: 1d
.ffifr-(2x-1)
C
dx

r-l|ll

-lilt

Visit For more Pdf's Books


Pdfbooksforum.com
)[,t]p

The Indeterminale
Fortrrs

llrtr chapter, we shall study two theorems whieh have


r.trcal importance in Calculus. We shall also liarn a
rrr,thod for finding the limit of a quotient of two
orpr.cially when such quotient can not be evaluated
)y tlr,' limit theorem (LO) in Chapter 1.
'l'ltt
orem

llrl:ort:m which we shall state below is a very useful


lh" ;,roof of many theorems in Calculus. This theorem
l n l r . r l b y Michel Rolle (French Mathematician, L652-

I r |IOLLE'S. THEOBEM

If a function f(x) is continuous in the closed


rrrtcrval [a, bJ; if f'(x) exists on the open interval
{rr, l)); and if f(a) = f(b) = 0, then there is a number
r rn (&,b) such that f'(c; =9.

*rrrrrtlhe proof of this theorem in this book. However,


,.f tlris theorem can be appreciated on the basis of its
r.rrrlr,n(:€.Consider the graph of y : f(x) in Fig. 5.1.
Trrt l,r'lween the points A(a, o) and B(b, o), there exisLs
!r{rr pornt on the curve where the tangent is horizontal
r ln F'ig. 5.1, the point in question is P. However,
,,,. nr()rc than one number in the open interval (a, b)
I r. I 0. I'hus in Fig. 5.2, the tangent line is horizontal
' , , t \ < : r .T h a t i s , f ( c 1) = f ' ( c 2 ) = 0 .

135

Visit For more Pdf's Books


Pdfbooksforum.com
13ti Differential and Integral Indeterminate Forms 137

v
TI must be a number c in (1, B) which satisfiesthe
conclusion'of the theorem. At, X = c, f,(c) = 2c - 4.
I But f'(c) = 0. Therefore, 2c - 4 = 0 and we getc =2.

i.2 Meirn Value Theorem

The mean ualue theorem (or law of the mean) is one of the
P f'(c)=O
rrrrportant theorems of Calculus. For instance, it is used to esti-

I
I
Y.f (r)
ttt;ttt) the values of functions when direct calculation is diffi-
r rrlt. It is also used to prove that two functions having.the same
--oi rh'rivative must differ by a constant. These are but only two of
rls rmportant uses.
: f
T(5.2) I{EAN VALUE THEOREM
t-
t
/i[
lfl

If a function f(x) is continuous on the closed


FIG. 5.1 interval [a, bl and if f'(x) exists on the open inter-
v val (a, b), then there is a number c in (a, b) such that
4
II f(L)-- f(a)
t

E(5.1) f'(c) =
b-a - '' t tI
I
'l
l

we shall use Rolle's theorem in the analytic proof of the


nrr';111 value theorem. Therefore, we must first form a function i
l't x ) which will satisfy the three conditions of Rolle.s theorem.
1,,'lt,hisfunction be given by the equation il
fil
- 1rl
-r o
I '#ii
'
-t, !i
i
'rrrl1' bhat
rh F(x) is ccntinuous on [a, b], differentiable {
anrl l,'(n) : F(b) - 0. Differentiating (1) wilr, rcspect m x ii
FIG. 5.2 t
b tr'(")={q=(4-r,(x) II
EXAMPLE. Consider the function f(x) : xt - (2\ I
the interval [1, 3]. Since f(x) is a polyn
\-- - it is contiiruous on []., 31 . Also f'(x) cc,t)flisiofl ul Roile's theorem. then
exists for all x in (1, 3). F'inally we nob t' xi sts a num ber c in ( a, b) such that F'(c) = 0. Hence, at
B : f(3) - 0. Hence the three conditions of ('quation (2) becomes
thesis of Rolle's thereorem are satisfied
#tl
Visit For more Pdf's Books
Pdfbooksforum.com
L.
138 Differential and Integral lndeterrninate Forms

F'(c) = {(PF {9) ' f'(s) = 0


b'-a lX r!\,lPLE1. Given f(x) - :<z + 2x - 1 and [0, lJ. Verify that
that hypothesis of the mean value theorem is
Find the value of c that sarisfies its con-
or equivalentlY :?:T,fj;:.
f(b) - f(a)
''' =
ff (c) Solution: f(x) - x2 + 2x - I is continuous on the closed
b--a
interval [0, 1l since it is a polynomial. We also note
which Wasto beiproved,
that f'(x) = 2x + 2 exists on the open interVal
The rnean'value' theorem may be ihterpreted geo (0, 1). Hence the hypothesis of the me€m i'alue
n Fig; S.3, we,9€ethat the'ratib tlaeorem is satisfied. To find the value of c which
H(o
b-a
will satisfy its conclusion, we proceed as follows:

Since f(x) - 1z + 2x - 1, o:1 and n=0, ,n"r,


is the slopeof the line throughA (a, ftal) and t (t, f 0, f(a)=f(l)--|
d(.) ir the slopeof the tangentof the curve y = f(x) at the f(b):f(0):2 '
p (c, f (c)). Thus the mean value theorem statesthat between the pr
Stnce f'(x) = 2x + 2, then when x: c, we hale
A ind B on the curvey y =f(x), there is a point where the tangentli f'(c)=2e+2
parellel to the line through A and B.
Substituting thesevaluesin E(5.1), we get
v 2+L
tI
2c+2=
1-0
I langent

lat
l1
t 5ecant
ancl c= th

P t" J(".i1
Ep
uil l \ \:\l l'LE 2. Use the mean value theorem to prove lhat
tb , i (
y=f(x) 7V1r.?< 10.85.
L0.7<
,t
r-l Grt
tll Solution:Let f(x) = \fx, a = 100 and b = !L7. Then f(a)
= f(100) = €00-= 10, f(t3)= f (11?)= r,fff7 ancl

sincef'(x) = + , then f'(c)' = :. Substi-


I1
2vx \/c
,) tut,rngthesevaluesrn Ll(5.1).we have
It 1 ,,ffi -10
il1
-l
2t/ c 11? - 100
)r
llll I FIG. 5.3

Visit For more Pdf's Books


Pdfbooksforum.com
Differential and Integral Calc rrrrinateForms L4L

,n may be used to approfirnate the value of a function


Solvingtor r{Iifliwe get - lr when b - a is sufficiently small.
tT
lrif t = F * 1 0 ;t Approximate the value of VTJ.

where 100 ,n: Let f(x) : 18, a : 81 and b : 82. Then


( 1 1 ) ' : L21, then it follows that f(a):l/Tf = 9
100< c( 121 f(b): ,tgZ
or 10< 16< rr f(x) = =L-
2^/V
Taking the reciprocal, we have
f(a)=
1 = 1
---
1 1\ 1 --5- '.t -
2JTI 18
ta' .vfr. 11 Srrbstituting these values in E(5.2), we get
Multiplying the inequality above bv then '/Tt -9+(82-81)
+and #
adding 10,
r7 * 10> 4- 77 _ 163 or 9.06
+10 18
20 z...fr
10.85> vTlT> 1 0 . 7 7 EXERCTSE5:1

tlurt the three conditions of the hypothesisof Rolle's


The inequality above is equivalent to
rntisfiedby the givenfunction on the indicatedinter-
t lrc value of c which satisfiesthe.conelusionof the
to.zi < /IT?'< 10.8b
which was to be proved.
flx) = x2 -x-2, [-L,2J
Now, supposewe write E(5.1) in the form f{x)-x3-3x, [0,tfST
f(b) = f(e) + (b - a) f(c) l{x) . xlnx , [0, 11
then consider again Fig. 5.3 Note that if b is near o, then o
and ftr) -sinx , [0rr]
udsonear o. That is, c comes closer and closer to o as the dll.
frnnceb-otgets smaller and smaller. Then we can see that whea r t,llowing, find c such that E(5.1) is satisfied..
b-nssufficiently small. c approximates the value of o. i.e., c g c.
Tftsynbol- is read "is approximately equal to". It follows thJf ftr) x2 ., [3,4J
f61*f'1a1.Replacingf (c) by f (a) in the equation above, we got
lr x ) v/F, [4, 9]
E(5.2) f (b) * f(a) + (b - a) f'(a) ft r ) . ' e x , [ 0 , 1 l

Visit For more Pdf's Books


Pdfbooksforum.com
L42 Differential and In rninateForms t48

*rt be evaluated by the methods mentioned above. The


8. r(x)= 1nx, f tlris section is to introduce a systematic method for
[+,+] tlr. limits of such functions. This systematic method
.i L'Hospital's Rule* and we shall abbreviate this as
Use the mean value theorem to prove each of the fol 'rrr convenience. This rule is stated here somewhat
I r' , without strictly mentioning the specific condi-
9. 2.1.L<{rg3 < 2.L2 l. 'fhe proof is also omitted here.
10. g < 2.o75
2.O7t < ^,/-+
I L'Hospital's Rule
11. 0.17( ln (1.2)< 0.20
If lim f(x) = lim g(x) = 0, then
Explain why the mean value theorem does not apply: X+a X+a

12. y = x213, [-1, lJ Iim9=ti*f,(*)


x*a g(x)
^*;g'(x)
13.y= ' [1,3l provided the latter limit exists.
x-2
)rrrrls,LHR states that fo eualuatethe limit of the frac-
Use E(5.2\ to approximate the value of each of the
frittt tale€sthe formf, x : e. differentiate the numera-
"t
+4. vB3- t,ntinator separately* and then' take the limtLof Lhe.
15. nrcf
", r:;+9 In case,this new fraction assumes
againthe
16. (2.03)4 {( x }
lh,. frrocossmay be repeated,that is, reapply LHR. How-
t7. w
rlapplying LHR at any stage, we m.ay simplify first
-)) 18. Qj2r3 r,lr*nt whenever possible.For instance,simplification . ,
!r t!
1rrr of common factors of the denominatot and
1e. w
5t
20. 1
rrrry be done first before reapplying LHR. The use of
)r rllrrstratedin the next three sections. I
G" 6 ,.tr.rminate
Formt$- a"a f
(tsr
5.3 L'Hospitat's3ule
r rtx) = lim g(x) = 0, then the function defined by
In section 1.4 of Chapter t, we learned how
(brr mit of a quotrent of +wo functions when the nu lrl to have the indeternoinate form 9 ut * = a. 'To
-1t norninator approach zero. In evaluating such t t)
changed it into a form to which the limit theoreme
L
-) That is, we employed certain algebraic"manip
Hi facforing an expression or rationalizing the l rt'nchmathematicianG.F.A. L'Hospital (1661-1704),who popular_
ever, thr)re are functions having indeterminate r tt'rtl"nokpublishedin 1696.
3tt
Visit For more Pdf's Books
Pdfbooksforum.com
\44 Differential and lnrcgral 'l'he
IndeterminateForms t45

evaluate
' lim I(-) = g , w€ apply LHR. Consider the EXERCISE5.2
*-' . g(x) 0
below llvaltrateeachof the following limits:
e2 t - cosx
EXAMPLE 1. Evaluate lim ,l . r:_- tanx - x
x+O u m
x - - +o x - S i n x
Solution: The quotient assumes
the fornr *n*n * - x2 * x.
f Z. lim xlnx
x--+1 (x-l),
Applying LHR, we get (1 - g* 1z
.,rr . r:-^
llm
.g2" - cosx = x-+ o xSinx
nm lim:-;
x-o x x* o I
x-s-ln{
4. lim
: 2eo * sinO x-+ o x2

*2(1)+ 0 ?
i).
,:
rUil
lncos2x
x-+ ?r (tr - x)'
=-t
t: 4"h* - 4
lr. llm
If the lim f(x) = lim g(x) = oo, then the fraction x --l ?I- lnSinX
X-*€ X*oo -
2
tr said to have the indeterminate form 3 ut x = a.
P
g(x) x - Arctanx
7. lim
applicable for this form x - - +O x2 tanx

1-vG-x
EXAMP LE 2. Evaluate lim -)(t- tl. lim
x-*F e2x x-+ 2 4-xz

Solution: Since lim -I' = co ' we aPPIYLHR' ll lim


xln (1 + x)
;
**- ei x+ O 1 - cosx
^
lim +=lim 2x ,. cosx
X*€ €z^
I t) llm
X*- 2e2* x *L sin2x
2
= -!im iL (cancel
X-r - g ,. lnxl o
ilm
\ rQ X

=tim 1
(reapply
x-* 2e2' ,tanx
t:l ilm
'
. f,tanSx
=0

Visit For more Pdf's Books


Pdfbooksforum.com
'fhe
Diffeiential and Inteenl calculue Indeterrninate Foms L47
L46

lim xcotx = x
a2x
lirn
13. Iim -- x+o x-ro ]/Cotx
x-+€ X"
= x . 0.
lrm
x -+ o tanx
(=T-)
x2
L4' IT* 1""
= 1
trm (by LHR)
g.x2 x* o sec2x
15. lim =T- + 1
x-*AX-L +X
=1
1n2x
(1)'
-A r.
Ib. llm
X--+ .oo X
=1
i
* sec2x
---
1/,
LI
lfm If lim f(x) and lim g(x) = oo, then the function defined
x _ f r s e c 2B x x-t a x+ &
2
bv f(x) - g(x) is said to have the indeterminate form - _ o o
1 - cosx ,
18. lim -- 'l'his
form can be, changed to the form
x+o x2 * oti uv algebric mani- U
prrlation so that LHR can be applied. I

- F
5.5 The Indeterminate Forms 0 (l o) and oo -
I:X,\MPLE 2. Evaluate liq (secx - tanx)
,rlr*f(x) = 0 andxrirn,g(x). = f€r then the function defined x-L
2

by the product of f(x) gi*ii. said'to have the indeterminate forn


into the foril
O.tt""l ut * = a. Formsbt tnit type can be changed Solution: This limit is of the type co - oo . since

- I or 3 so that LHR can be applied. To effect this change, we re' = - drldt*i, =€


(, ao tu"
I
write f(x) e(x) into any of the following forms: |
- f(x)
or
s(x) . / -
1 sinx\
1/f(x) lim (secx- tanx) = lim I (why?)
) 1/g(x) x-L x- lL \COSX CqSX/
artl 2Z

I
In general, one ,form is better than the other but the choice wi['
dedend upon the given product f(x) g(x). =rim i-'it* ,, = o
x-* lL cosx 0) i
2
lli
EXAMPLE 1. Evaluate xcotx
lln
1)
rj* 0 - cosx
Solution: Let f(x) = x and g(x) = cotx. The limit of their : rgtt
- SlnX
(by LHR)
x--!
-) product is of the tYPe 0 - since f(x) = 0 and g(x) 2'
tllrl
rf$ ,=@ atx=0.ThuS

etl
Visit For more Pdf's Books
Pdfbooksforum.com
tr:rminate Forms 149
148 Differential and Integral

= lim cotx
'x*4.
2 1\
/ x -
= c o 0.;T l [Ft ,"")
a
/ 2
1 -;i"';/ \
\1 - cosx
-0
G --r*\
EXERCISE5.3 \ x sinx/

Evaluatepach of t*re following limits: / L -. 1 \


\rn 1f + x) Arctanx/
/t- \
1. Iim (e'-1J*
x*@ \ /

2. lim sinxlnx lndeterminate Forms 0o, 1- , and *o


x-O
function defined by the expression f(x)etx) may, at a
nx vnlue of x, assume any of the following indeterminate
B. 1i6 -3f 6 n
x--+ox 2

4-
- r' [n \
tanx oo if f(x) = 0 and g(x) = 0
l'+ (t-,
l'" if f(x) = 1 and g(x) = €
5. lim *rir, I
X+oo X '''" if f1x; = ". andg(x) = o
)
I
6. lim (4 - *'1 tan* $ nr**\* f(x)rtxl when any of the indeterminateforms above
x+z 4
) l, we ma Perform the following steps;
t 7 . lim xcsc2x
p
x-O I l,r,t N = lim f(x)st*l
- 'l'uke
the natural logarithm of both sides of the equation
8: lim (cscx - cotx) r r r( 1 )
x* O
t = ln lim f(x)s<xl
e' I'T,(;-* -#)
ln N
x"+ a
)
= lim 1nf(x)r{xr by L12
- x-+ a
I
t 10.*r,T,(#r;t)
= lim g(x) lnf(x) by P3
fl
il
x-r a

Visit For more Pdf's Books


Pdfbooksforum.com
1.60 Differential pnd Intesdl rminate Foms 151

The limit at this point takes'the form O' (


yb, therefore" apply tlre method used in
Thus

ln N = .}H8 rhis limit is or the


**(d
3. 2:,
J31 1+0

4. APply LHR to the right member of (3).

6. Suppose ln N : f.1.Then Jrf = eL where t


number. Therefore

f(x)rtxr= eL
lY"
rim (r*a) = e2
r.@ x
lio, ( 1*3\' \ /
EXAMPLE. Evaluate xx**\ I
EXARCISE5.4

Solution; This limit is of the type 1T eachof the following limits:


3
1r+ zxj.
L e t N = l i r 4f L *'.x? y
x*€\ /
(4x - 1)x

Then lnN = lim 1n (t*3).


t *** \ X/
h (sinx)*

t =limxln /r.g\ X/ (=-.0)


x-)€ \
I
(cscxltio*
ril

t
t=3l xl
-
1.
x

i
(tanx)G
t

rl (by LHR) ^/ r4x

st (t.a\
rl
*/
\
ill
Il

Visit For more Pdf's Books


Pdfbooksforum.com
L52 Differentnl and u)t,(;p
-x
8 . lim xe
X-OO

1
9 . lim (x + cosx)r
x*0
The Differential
10. lim (1 + sinx)cot"
x*O
t^r we have regarded ttre notation-$f, as a single symbol
11..lim (t*3x-r). far,t. the limit of the quotient * . !0, Uor
lbvv^vrru *A X eur
u nx
o.L r to
L \ , | Irepresent
.y|Jfe5gllU

rlcrn,rrtiveof the function y = t(x).Nqw we shall introduce


I llf r,'1111'fysimple but useful concept ealled lhe differential.
t2. lim (1 + tanx)T fil'\ <:oncept rvill give meaning to the symbols dy
x*O .and dx.'
lr'lv and in effect will perniit us to cbnsider the symbol
x4

1 3 . lim h - lf rr, rluotient of two differentials.


x+ o\ "rF/ f )t f fi.rr.n tial : Definition and Interpretation

L 4 . lim (sin2x) tan2x 'r,rrsider


f a function defined by y - f(x) where x is the in-
x-{-
4 tfrrl*rrt. variable. In Chapter 2, we introduced the symboll
lr, rk'r'rotethe increment of x. Now we introduce the symboi
rlrr,'lr we call the differential of x. Similarly, w€ shall call the
(1 - 'cotx
15. lim *2 ; ,l r1,,,as the differential of y. To give'separatemeaningsto
x-r O lrul rly, we shall adopt the following definitions of a function
I lrV the equation y = f(x).
< I r l , l , ' I N I T I O N6 . 1 dx =. Ax #
f't
r-r
frr w<rrds,Definition 6.1 simply says Lhat the differential
-JJ,t, fir i,t1lepbndentuariable is equal to'the increment of tJteue-
lEl

dy = f'(x) dx
I ) } : F ' I N I T I O N6 . 2
il
-rr)

-Jss'
f rr rv<rrds,Definition 6.2 statesthat the dlfferentiol of a func-
tr t'qual its deriuatiue rnultiplied by the differential of its
{#
t . .--r)r ,,rr,ttt,ntuarigble $
l+
GI

GI 153
fi
#
qf
tu,

Visit For more Pdf's Books


Pdfbooksforum.com
h\
L54 Differential and Integral I lrr. Differential 155

We emphasize that the differential dx is also an in v


dent variable, i.€., it may be assigned Any value wha
Therefore, from Definition 6.2, we see that the differential
is a function of two independent variables, x and dx. It
also be noted that while dx : Ax, dy * Ay in general*.
y=f(x)
Suppose dx * 0 and we divide both sides of the equa
dY. = f'(x) ck Tangent

--t- ay
by dx. Then we get 4v
= f'(x)
*

Note that this time $f Oenotesthe quotient of two differe


i.e., dy and dx. Thus the definition of the differential mc
possible to define the derivative of a function as the ratio of
differentials. That is,
dY - differential of Y
-f (x)
\--l = dx
differential of x FIG, 6.1

The differentiai may bc given a geometric interpi From analytic geometry, we know that

<
Consider again the equation y - f(x) and let its graph be-asslt
in Fig. 6.1 Let P(x, y) and Q(x * Ax; y + Ay) be two pointr sloPe of PT = ;;an 0 'l',
the curve. Draw the tangent to the cune at P. Through Q,
a perpendicular to the xra:<is and intersecting the tangent at lfrrt rn triangle PRT, we see that
bt
Then draw a line through P, parallel to the x-axis and in RT RT
tanO :;; (2)
the perpendicular through Q at R. I,et O be the'inclination of
G, tangent PT.
Err ll,wr.V€r,Ax = dx by Definition 6.1. Hence (2) becomes

tan0 = RT (9)
tsrt F
JrP'
7r,,rn Chapter 2, recall that the value of the derivative of y = f(x)
\er rt l' is equal to the slope of the tangent at that same point P
gii *Recallthat Ay = f(x *Ax)r- f1v). l l r . n c ei n F i g . 6 . 1 ,

d,ll
Visit For more Pdf's Books
Pdfbooksforum.com
157
156 Differenti'd and lntegral

sloPe of PT = f'(x) d(un) ='nut- I du

Substituting (b) and (4) in (1), we get


=fril
d(\au)
f'(x)= H -,
d l/:\ = d.',
rn+r
\""/
Solving for RT in (5), we obtain
rYrrrlrol d is regarded as the opqrgtor which indicates
RT = f'(x)dx ,,f finding the differential of a function'

But by Definition 6.2, the right member of (6) is dy' ft.rr.ntiation formulas for other tvpes of futrcbions
trr Ohapter 4 may alsorbe expressedin terms of diffe- '
RT-dY

we see that dy is the increment of the ordinate of the l'ind dy if (1) V = x3 '-' 4x2*.5x and(2) v: #n
Ay ir
line corrpsponding to an increment of Ax in x whereas
corresponding inerement of the curvenfor the tT:^i ,
f'tx)I gives the
in x. We also note that the derivitl":#.9I ot S
nf
(1) y =x3-4x2+5x
the
if tn" tungent white the differential dflives the rise tl

gent line dy:d(xt -4x2 +5x)

6. 2 Differential Formulas : 3x2 dx - Sxdx + 5dx


Since we have alreqdy consider"d * as the ratio of twO
- (3x' - 8x + 5)dx
f"r"rrli.tr;ttt"" the differentiation formutis in Chaptnr2,(S-99'I
may now be expressed in terms of differentials by multi
- both sides of the equations by dx. Thus (2) y =
3*31
-r
d(c) : g dy = d /-2.-\
G,,
dl.
(s,.-t
hrrr d2. d(x) = dx (3x.- 1) (zdx) - (2x) (3d.x)
(3x - 1)2
d3. d(cu) = cdu
-r;t (6x-2)dx-6xdx
d4. d(u+v) = du + dv
nl (3x - 1)2
tJ
I d5. d(uv ) = u d v * v d u
I
"--- 'qa

vdu - udv - 6x)dx


.FqilI d6. d$) =<+

v2
=6(6x -2
.dlil
Visit For more Pdf's Books
Pdfbooksforum.com
r-
158 Differential and
159

- ?dx EXERCISE6.1
=
(3x - 1;z
,rrrnplify wheneverpossible.
Note: In practice, we simply get
of the right member of the !'
'3xa-4fr*2x
multiply it by dx. Thus for
lem, the solution will si v ffi
as follows:
y = x3 -4;z +5x v 4eZx

dy = (3x'-8x+5 v = x -32x _
EXAMPLE Z. Find tUn meansof differentialsif
dx v xlnx
$olution: xy * sinx : lny v *r
"-2x
xdy+ydx*cosxdx:idt v *x + fsin2x

!' - excos2x ' - "a

xydy *- yt dx * ycosxdx - dy 3x sin2x


v sin4x
*
.dv.
*Ya*+y'+ycosx=fr ., dv T- l- Bz
y Arctan (tan3x)
dy- 9I =-y2-ycosx
- xy _ax
ox }7 use of differentials: .!i .i
-q fl
ll

-.4 (xy- 1)
*{:
-y (y + cosx) xy + Arctan (xy) = 0 I
r{
-atrl
,l
dy = -v (v +.qol9- :tln (x2 * y, ) : Arct*
* I
It
dx xY-1
lh1;p
r' I xy, - y
dI
tr
(- ,.\' = sin (x - V)
\_
-s[ lx t 4Y: 32

lrrx l. Zlny: xy
-rHili
Visit For more Pdf's Books
Pdfbooksforum.com
L.
'l'he
160 Differential and I Differential 161

6.3 Applications of the differential


Note that if we have been asked to find \/7, or lffi, then
rtur opproximation by use of differentials would not have been so
Let us copsider again the.graph of y = f(x) in Fis.
sood.Why?
that the difference Ay-dy is represented by the di
ment TQ. The ma'Initude of this difference can made as r.rxAMPLE
2. 3, find theapproximate
please by making Ax sufficiently small. In other word8, $; m; J$;. latue
{*x, we expect dy and Ay to be nearly equal. We are
effect, that dy may be used to approximate the value ot solution: The exact value is y * Ay but since we are simply
while the true or exact value of y at x * Ax is asked to find the approximate value, then we
shall solve for y + dy. Note that if we write
f(x+Ax):y+Ay, Z.AL:2 + 0.01, then we are considering2.01 as a

the result of applying an increment of Ax : dx =


its approximate value for small Ax is 0.01 to an original value of x : 2.

f(x+Ax)-v+iy, Since y = x3 + 2x2 -B (1)


f

where (recall chapter 5) the symbol ":" is read "is then dy - (3x, + 4x)dx (2)
equal to".
t
When x: 2, then from (1)
EXAMPLE 1. Compute lF approxirnately by u
tials. V:8+ 8-3:13

Solution: Let t/ffi: y + dy and when x : 2 and,dx : 0.01, then from (2),

and 37:x*dx d y : ( 1 2 + 8 ) ( 0 . 0 1 1= 0 . 2 0
where x is a perfect square nearest to
€' Therefore, the required approximation is
Obviously 37 : 36 + 1'
y+dy-13+0.20=13.20
x = S6anddx = 1
\r
Hence
<, Let y: /r I';XAMPLE 3. Each side of a square is increasedby Ax. Find the
approximate and tme increase of the area A of
dx
hr
Then dy = t;F the square.

Solution: The approximate increase in A is dA and'the


hr For x = 36, v: 1ffi-( true increase is AA. The area of the original
1 t square(ABCD in Fig. 6.2) is
- dy= =-
we have
2\re6 12
\ra
A: x2
- Therefore, tF-:-6+ _1 = lg
12 1l
G
Visit For more Pdf's Books
Pdfbooksforum.com
tdz Differential arrct Integfal Caleulu 't'he
Differential 163

By differentials,
The differentials ,ar-.ealso used in approximate computation
dA: 2x Ax = 2xdx of certain quantities due to small errors in measurement. For
if y = f(x); then an error dx in the nteasurementof x leads
rrrstance,
Thus Zxdx gives the approximate increasein lo ah approiimate error dy in the quantity y. The approximate
In Fig. 6.2, this approximate increase is
rr.lativeerror (RE) in y is the ratio d v and the approximate per-
by' ;
'',,ntageerror (PE) in y is.dJ (100). -]r

'dA _ area of DCEF + area of AD


The true'increasein A is l:xAMPLE 4. The radius of a circle is measuredto u.ero cm with
a+ error of 0.05 cm. Find the relative emor in the
AA:(x+Ax)r_x2 computed area,
:
_xr+2xAx+(Ax)2-x2 Solution: We are asked to find the RE in the areb A when
r: 10 cm and dr: 0.05 cm. Hence
: 2x Ax * (Ax),
A:n t2 :n(10)2 : 1002rcm2 I
In Fig. 6.2, this true increasein A is AA =
DCEF * area of ADHI * area of DFGH. dA = 2n rdr : 2n (10) (0.05) : r cm2 r
-.:t
Therefore,

RE=#

1002r . . I
il
= 0.01
tb

};XNMPLE 5. Find the approximate percentage error in the


G computed volume V of a cube of .edge x cm if an
-t error of 2% is made in measuring an edge.

Solution: We are asked to find the PE in V when the PE


b
inxis*ttoo)= 2vo.
.J

f
\- Since V: xl
G, FIG. 6.2 then dV - 3x2dx
4,
Visit For more Pdf's Books
Pdfbooksforum.com
L64 Differential and Integral CALCULUS I lrfferential 165

tlr,. following problemsby use of differentials:


and the percentageerror in the volume V is
I lr,' circumference of a circle is 100 cm. If the radius is
pE = troot lnt'roasedby 0.1 cm, find the approximate increasein the
$ ltr.it.

_3 x 2d x ( 1 0 0 )
fl rrn error of L.|o/o is made in measuring the side of an
x3\'
rrlrrilateral triangle, find the percentage error made in the
ronrltuted area.
= r(q:,100)
'f'lrc
radius of a sphere is measured to ,be 4 cm with an error
of ().002 cm. Find the relative error in the computed volume.
: 3 (2%)
hr rr right cirgular cone, the radius of the base is half as long
= 6c,6 l t lre altitude. If an error of 2% is made in measuring the
flrlrus, find the percentage error made in the cornputed vo-
.l uttt,' .
EXERCISE6.2
ltlrrl the approximate surface area of a sphere of radius
I u2 in.
Find the approximate value of the following by use of differentials
-
?lrrtl the approxiinate area of ,a square when the side
1. \@6
| {)l cm.
2. :ffi l ,'ircular hole 4 inches in diameter and 1 foot deep in a
bh,r'k of iron is drilled out to increase its diameter to 4.1
3. $Z1tt+
ln I'ind the approximate volurne of the rnetal removed.
4. (63.4)2/3
Tlrl diameter of a circle is to be measured and its area com-
prlr,rl. If the diameter can be measured with a maximum
br 5. ffi6
firor of 0.002 cm. and the allowable error in the. area is
3 O (ll uh2 , find the diameter of the largest circle for which
6. (1.98)4
ll',. sneeifications are met.
(br
7. ln (2.3) if In2 = 0.6931
],,r' a ri ght ci rcul ar cylinder of height 25 cf f i. , t he r adius
rns measured as 20 cm with an error of. 0.05 cm. Find
hr 8. if.e2 : 7.3891
"2'4 lll' :rpproximate percentage error in the computed volume.

a
\,
Use differentials to find the approximate value of y:

9., = ( 2 x - 1 ) o w h e nx : 0 . 9 8
tl
10. y=x4-2xr +3x2+x-1 whenx=1.02
Jl
Visit For more Pdf's Books
Pdfbooksforum.com
x,ep

rnnlytic geometry, w€ have learned that a curve may also


x.(l analytically by a pair of equations of the form

x : g(t) , V: h(t).

rrtions arc called parametric equations of the curve and


I variable f is called a parameter. For example, the circle
- n2 may be represented by ttre parametric equations

x:acost, V=aSint

l)arameter t is the angle between the x-ilds and the


tlrepoint (x, y).

ives in Parametric Form {

!' f(x) be a function whose parametric representation


It the form

x:g(t),Y=h(t).

rrrChapter3 (seesec.3.7) that


l)
'll
dx =
rate of change of x with respect to t
ter
ai

ty = rate of chang.eof y with respect to t


t)l
l*r
dt
ln*r

rrtly the rate of change of y with respect to x of a fun<i


rrr
rl lry x = g(t), y = h(t) will be given by

)tt dy

,-t
E(7.1)
4y=
dx
_r
dx
l' rnrttt dt
iltt
Iilll
t67
Visit For more Pdf's Books
Pdfbooksforum.com
168 Differential and Integral f)erivatives from ParametricEquations, 169
Radiusand Centerof Cuwatute

Next, we consider the problem of finding the sec d*- = dY =


d,cosg 4sing
tive of a function defined by the parametric equations d0 d0
Chapter 2 (sec.2.7), the seeondderivative is defined as
dy dy lde
Then
&v dx
= d 1dv\ dx/d9
d.: d* \d"/
= 4tt19
and by the Chain Rule, we may write the eqIual
ati,
lon 2cos0
form
\ = 2tan0
dty. = g dy' :-l
E(7.2) dt ( d"
dx2 gt v - d
and 121ora&
oy
dy dx' de dx
EXAMPLE i. If x = t' - 1' , y = t 7 + t , f i r rd
dx
and
1
= Zsecz
t
ffi
Solution: Since x.: t3 - I , Y= t2 +
= sec3g
' dNt: 2 1 +
dX - .t+2
then eJ t/

dt
EXERCISBZ.r
B y E ( ? . 1 ), dv - dzv
l " rnd I' and +- and simplify whenever possible.
, dx d xz
- q y= Z t + I
dx gt2
,- x=
I
1r ,r 1, y = t2 + 1
< and by E(7 .2) , ,)
x = t-3, y = t3 + gt
a

: .i
bu d'v =g ?t:1\dt
dx2 dt gtt ) d* :f. x = u3 + 1 , V = 4u2 -4u
4, - (2t + 1) (6t)
_ ( 3t') (2) ,t. x* t/ffi,V='u2-3
hr
9t4

*?(t:tl l-r. X: 1* cost , Y = sin2t


b- =
9ts (i. x- 1-lnt,Y=t-lnt
- 'i
EXAMPLE2. If x =2sin0, V=1-4cosg, find
dy X_ cos# + tt sintl, V T sin0 - Ocos0
\bu
dx
rr!t t1 x= cos34 V: sin39
Solution: x=2sin€ , Y=1-4cos0
-= 11. x= e Q ', y = 2 e - 6

Visit For more Pdf's Books


Pdfbooksforum.com
170 Diffenential and'Integral Calcultu I )erivativesfrom ParametricEquations, 1.71
Itadiusand Center of Curvature

10. x= Os[,,V=g0
k'^gth from a fixed point A to a variable point p(x, y) be
denoted
= y = a (1 - c osO, l,y the srnall letter s (Fig. 7 .l). consider a nearby p"int
1 1 . F i n d t h e sslope of the cycloid x a(g- sing), e(*l+ ax,
TI v + Ay) and'let As be the arc pe. since s is a function of t, then
when 0 = 2 ' rv(' may wish to find the rate at which s changes
with respect to
= ds.
L2. Find the sslope of the curve x - e0 sinO , Y e0 cos9 w hen
t ia

f
" ""''dt

a = aT
Ity definition, ds rr --A s
= 2 iint, aF=A lun (1)
13. l'ind the equation of the tangent to the curve N t*o At"
= ; .lt

L4
Y = cos2t rw h e n t

Find the requation of the 0argentito the curve x


: lnt 'V
lltrt we may express A s
ZT U'
f
rel)t = fl.

flil
: t-l whe
As = EA 'sA TP Q (2)
Tt
T.2Differentialil of Arc Length
urve be defined parametric4lly by the equa
t
Let a cu wht:re the chord PQ in Fig. 7.1 is the hypotenuse of the right
x:B(t), y=h(t)

'ivhere g and h are differentiable functions of f' Also let th


the
t rianglePLQ. We note that

PQ= (Ax)' (Av)'


I

(3r x
v
4I l'hen (2) may be written in the form
li
II As _4s
<l II At PQ
r
At a

.l
ff
,illt
I As - As-
hrr
I ,I\
t
angent
1-Tansen
\z
or
At PQ
(4) ffi
alt
frq1
I
I .Q/ - ,l[rt
I Srrbstituting(4) in (1), we get iilq
b;,

i -.-A e4:- jl = lim As 'Ax '.(#)' ll*


ffrr
dt ar -'o PQ At
(51
j
As
ar\' * /4.I\ '
/Ax\ 2

ff
,nl = rrm
----_--n- o'lT. PQ /, I n*-/
At/ atl \
vr I
tr,
I
et I be shown that the lirnit of A s as At , 0 is unity, that is,
A s . ' 1 . H e n c e( 5 ) b e c o r n e sF6
/tf,
iffin
I
pq
rgti

{
I nrlt
F I G . './' . J' .

Visit For more Pdf's Books


Pdfbooksforum.com
L72 'Differential qnd
Integral Calculur lves from Pararnetric,Equations, 173
nnd Center of Cunraturr

ds
aT- (*)
2
.(#)' Irf r' <rf change of O with respect to s is called the curuature
lrrrye at P and is denoted by K. That is

Multiplying both sidesof (6) by dt, we obtain K=qd


as
E(7.3) ds = (#)'. (#)' Irlu. of K is either positive or negative. If K > 0, the curve
)'nv. upward at P. on thc. other trano. if K < 0, the i,
where ds denotes the differential of arc length. From E(?.8), * rlownward at P. However, it is customary ",r*"
to consider K.
can obtain the fo'llowing forms for ds: llrve. For this reason, we rewrite our defining
equation
E(7.4) d s = V ( d x ) 2 + (dv
E(2.8) K= l-qEl
E(7.5) ds= (vJ dx ld'l

E(7.6) 1. (*i) dy
v
ds=

If the equatiorl of the curye is given in the polar form r : f (g) -


then the differential of arc length is given by

E(7.7) can easily be obtained by use of the familiar relationl


Tangent
between rectangular and polar coordinates, that is,
br
x = rcos9and y = rsinO.
a, ?.3 Radius of Cunrature
-r
We have seen in our previous discussion, that the conce
of the derivative is related to the tangent to a curye.. Anothe!
- concept of g-eometricinterest is that of curuature. Consider tht
curye y - f(x) as shown in Fig. 7.2. Let s be the length of the a.no FIG. 7.2
F
of the curve "between a fixed point A and a variable point p,
- Denote the slope-angls* of the tangent T to the cunre at P by 0. 'ltr*
reciprocal of the curvature. is called the radius of cur-
€, I rrrtl is dencrtedby R. That is
rThe angle between tho tangent and the x-axix.

a, R=*

Visit For more Pdf's Books


Pdfbooksforum.com
Differential and Integral Calculus
Derivativesfrom ParametricEquations'
Radius and CenGr of Cunrature

the expression for K in E(?.8), we have the defining formula


(2) *U we get
Substituting (3) andr(5) in

' ds =L'
|,+/gr\'l-''
\d*/ J
lTlifving'
E(7.e) R=ls
dd dty
dx? next derive a formula for R which is applicable if the
of the curve is given in the form y f(x). By the chain
we obtain
Substituting (6) in S(?'9)'
r t3l2 ds =ds dx (1)
d0 dx d0
p
ru =
[: :-(fJ'l
a
d ' 'vy
I I tion (1) may be written in the form
rt ds - ds/dx
(2\ .
d0 d@/dx
not needed in the
The absolute value symbol is
w€ may write (7) in
/ av \ z ;' 0. To simplify notation'
'I

l:(7.5), we obtain
\dx./ ? -rv -.._t

E(7.10) R = [t i [y']'Jn
lv" l ds
dx
r-
(3)
d'Y
whereY' -- fr *o Y = AF
dv ,'--,, . rlcfinition of slope,

tan$ = g
is given by x - g(y)' dx
4:, If the equation of the i*-"
for:p
fining equation for R takes the
lh-.,

E(7.11) dy (4)
<" O = Atctan dx
brr, -,d>A2x =
where x = qy
,: and x" = rtiating (4) with resPectto x,
dy'
dry (5)
b,

/t)
form
When the equation of a curve is
given
ar ;Ts
d0
Ao
dx'
dx2
/ A.,\ 2

\rr,
x:B(t),y=h(t)
-ii
G),,
Visit For more Pdf's Books
Pdfbooksforum.com
176 Differential and rtivesfrom,ParametricEquations,
s and Centerof Cunrature t77

the radius of cufvature can be shown to be v


(g'\' + (ht
E(7.12) R =
g'h" -

dx g"=#,h'=#,&d
where g' = -
dt'

r
$
t It'can also be shown that the radius of
curt€ r = f(9) is given by
t
-,
E ( ? . 1 3) R= & G,I
ii"
s
&
w
12+ (r')
I
t
!' d2r
fi
wheret' = * *O t" =3'! . The proof of E(?.lS) L
i. # de2
cise. However, w€ shall state and prove a theorem
grving the proof of E(7.13).

THE'REM' tr'IG.7.3
ff fi"'H,ilf,f,:"1ff"1*: a = 0-e (1)
point P(r, 0), then
hlkrws from (1, rhat
< E(?.14) tana=i
tana = 9n0-tane
Er 'dr r =
w lnere 1 + tanOtang iz) "
do
G lrom Fig. ?.3 and by the definition of slope, we have
Frh We shall now give the proof of the
be the angle between the x -a:<is and the radiul tang = Y (3)
x
d the inclination of the tangent to the cune at I
Frrn between the radius vector and the tangent (seo
e.0, and a are related by tne equation tan 4t
qx (4)

C
r€l
rtuting (3) and (4) in (2) and simplifying

tan q = *dryd* (5)


xdx + ydV
rAi:,i,
Visit For more Pdf's Books
Pdfbooksforum.com
!.7,8 Differential and Integral | )r.rivativesfrom Parametric Equations, L79
Itndius and Center of Curvature

From analytic geometry, w€ know that


x: cost v' - - 2sint
x=rcos0rV=rsing ,, = -sln
x ty" : - 2cost
By differentiation, we obtain from (6) the following eq
fi
When L _

u'o
dx = -rsingOb + cosgdr, dy = reos0+ singdr Zl

Substituting (6) and (?) in (5), we obtain


=0 y ,= -2
t
. rdO x
f,an q =
ldf
t,
x" -l t
y= 0
or' tanq =*
d0 Substituting v a l u e si n E ( 7 . L 2 1 ,w e g e t R : 4.

-d1
Letting r' = , w€ obtainE(7.14).
# EXERCISE 7.2

EXAMPLE 1. Find the radius of cunrature of y : ;r )'tnd the radius of curvature at the given point:

Solution: Since y: *t, /x .r\


t . -v = - g { . E * s - ? / r t ( 0 , a )
2\ /
then Y'= 3x2 and.Y"= 6x
OS
\,lr whenx:1, y'=3 ?Ddy"*6 v xa at (1, 1)

rEtt Then by E(?.10), v sinxat (t, tl


(A)zlgl
R _ [1+
lhl
-||] 8
1 6i x = at (2,0)
-rt
,4 n
5y'm x = sr -Zyat (1,0)
3
Dtt
yz = 4x at (L, Z)
l,lr
EXAMPLE 2. Find the radiusof cunraturepf the cuwe x = 16x2 + 25y2 - 400 at one end of the minor a:<is
lil
-1,V=2cost+ Satt -!.
z x=etry=e-tatt=0
3r
{tl

dlt,
-lttr
Solution: x=sint-1 V: 2cost+ 3 x=tl-2t,V=L-4t at t=1

Visit For more Pdf's Books


Pdfbooksforum.com
180 Differential and trntegralC lvot from Parametric Equations,
Irrl Center of Curyature 181

10. x=2sint,V =cos2t atf,= tl v


6

11. x = etsint , V = et cost at t: 0


1 2 . r = a c o s 3 0 a t0 : 4
6
'13.
r=a(l-cosQat 0=r

Find the radius of cunature at any point on


the curve.
t4. y = lnsinx

--"1
'1S.
x2l3+y.213-,213

16. V2:8x

77. b2x2 * azy2 - a2b2

18. t2 = az coshl
FIG. 7.4
19. x = cos@+psinf , V =:tnp:@eos@
r,rtruationof the circle of cunrature is
20. x2+y2=az
(x- h)? + (y- k)2 : R'
2!. Derive E(7.12) (1)
r n gE ( ? . 1 0 )i n ( 1 ) , w e h a v e
Itr 22. Derive fi(?.13)
(x**h)'? + (V-t<;' - [1+(Y')1 ]3
1: 7 .4 Cents of Cunrattrre
(t'tr- Q')
-l
Through any point P(x, y) on the cunre y = f(x), we * slope of the tangent at p is y', then the slope of the nor-
construct a tangent.circle whose radius r'is equal to the | ,,v'.The equation of the norriral is
-l
of cunrature R of the curve at P as shown in Fig. ?.4. This
Jt
circle is called the circle of curuature and its center is called v k=-+(x-h) (3)
center of cuntature of the curve. This center of curvature v
hascoordinates(h, k) lies on the normal to the cune at p. we

Zi show how the coordinates (h, k) can be expnessedin terms of


coordinates (x, y) of P.
| 2 ) and (3) simultaneously for h and k, we obtain

+
A" E(7.15) h = x - V'[1 (Y')zJ
ytt

Visit For more Pdf's Books


Pdfbooksforum.com
182 Differential and In

, I + (y,)2 Partfal
E(7.16) t(=y+---
v
Differentiation
EXAMPI- E. Fincl U're eenter of cunrature of the grreceding chapters have been concerned with the dif-
at the poin! ( o) , s of functions with one independent variable. In this
""
we shall stucty differentiation of functions of several
Solution: Since ' =e-x
Y l)xamples of these functions are the following familiar
t -g-x
then Y'
V = rrr2h (1)
Y" = b-r
A- | absin6 (2)
At (f'0) Y' = -1
( t ) expresses the volume V of a right circular cylinder
y" = 1 of the base radius r and the altitude or height h. That
I function of two variabbles, r hnd h. Formula (2) shows
B v E ( 7 - 1 5 ), h-/ uea A of an oblique triangle is a function of three vE-ria-
y a, b, and 0.
BvE(7.16), k:3 -
Derivafive
Hencethe center of curvatureis (2,
,, := f(x, y) be a function of two independent variables
EXEBCISE?.3 . lf y is held constant, then z becomes temporarily a func-
lhe single v,ariable x. From thls point of view, we can
the derivative of z .with respect to x by employing
<: Find the center of curvatureat the point indicated.
f.r ordinary differentiation of functions with single
'l'he
derivative found in this manner is called the pqrtial
1. y = * (". + e-.*) at (0, 11 5. v = lnx at
lbrrl 'f z with respect to x and the processinvorved is called
'ferentiation.
6. y ={tan2l The derivative of z,with respect to x is de-
t',: b.y= slnx at6 ,1) nny of the following symbols:
-rrr 3.y2= 8x at (+ t) ;)f . ,
zx, f" (x,v) , f*
;;;'
4.xy= 4 at (2, 2)
hrrp

rf x is held constant, then z becomes temporarily a


'f y. As a result, we can compute the partial deriva-
\rr-r with respeet to y and this derivative may be denoted by
-i following symbols:
183
tG,,:,
Visit For more Pdf's Books
Pdfbooksforum.com
: 185
Differential and In l'artial Differentiation
184

+ Ay)
and f(x, y.* Ay) = xt (y + Ay) + 4x* 3(v
dz af zy , f, (x,Y) , fy
6ly ' T'y , By E(8.1).

tot ffl
It shouldbe noted that the svmbol# = lim
Ar-- o
thoughtofasafractionsinceneitherofthesymbqlg
2 x y A x + ( A x ) 2 Y+ 4 A x
meaning*' The symbol = lim
(or dz and 0y) has a separate Ax -' tl Ax
with respect to x wh
means to differentiate partially
a
it. The symbol it interpreted in like manner' - lim (2xY*'Ax.V + 4)
fi
Ax' o
Foimally, the definition of partial derivatives
:2xy+4
as follows:
partial derivative of z withr Ily E(8.2)
lf z : f(x, y), then the
- [x'Y+4x"+3YJ
x is sYmbolicallY defined as *
n2 = lim [ *'(y + AV)* 4"_. tjY ALI]
ily Ay'j o Ay - ' , i i

x2AY + 3AY
= lim
with respect to y is AY
and the'partial derivativ e of z Av; o

E(8.2) yd y = lim IG'-vt-AYE-I(.'v) - lim (x2 + 3)


Il
Ay*o Ay AY. o 1I
<l This definition can be exten{ed t9
tu1'ctions of -x2+3
fr.

"1
I

function f of severl
two variables. in g"ttttul, with with ln piactice, we c<lmPute;Y tn in the examPle above bY consi' I
lr-al
.tnJ* is a partial derlvative 0x
;:;,';:':--*-". f,
inalir""aent variable,i'e', f* ' fy ' rlcring y as constant and then differentiate with respect to x by
Gi:, rrpplying the rules"for ordinary differentiation'
*o Ot
l!r-rr EXAMpLE1 f z =xzy + 4x +3v, find# # 'l'hus
E(S'1) and E(8'2) resPectivelY'
: x2 Y * 4x + 3Y Since C= x2 y + 4x + 3Y
lfr- rl Solution: Let z: f(x,Y)
tl
Then f(x + Ax, Y) = (x + Ax)2 Y
+ 4(x Then = (x'Y + 4x + 3Y)
# u?
L
"--ll

letter deltr
*The symbol 0 which is a special form of the Greek -tvY*4
-y,
by'Jacobi(180+1851)'
ll
i''
l--
Visit For more Pdf's Books
:
E
Pdfbooksforum.com
I
186 Differential and Integral C I'artial Differentiation 187

To find P. *" treat x as constant and differentiate with It shotrld be noted that before performing any partial diffe-
dy rentiation of functions of several variable, it is important to know
to y. Thus first which of'the variables are considered or held constants.
dz = 8.2 Geometric Interpretation of purti"t Derivative
+ 4 x +3 y )
0y #a"'y
We shall now give a simple geometric interpretation to the
:x2+3 concept of partial derivative. Let the graph of a surface* defined
lry the equation z = f(x, y) be as shown in Fig. 8.1. Let P(x^,
Vs, 26) be a poirit on the surface. I'hen the plane passingthrouglr
EXAMPLE 2. If z = xsiny + ysinx, find P'arrd 31. P and parallel to the xz plane has the equation V : yo. The inter-
dydx
section of the surface z = f(x, y) and the plane y : vo-is the curve
APB as shown in Fig. 8.1. As a point moves along the cunre APB,
' Considering y as constant and differentia its coordinates x and z vary while y remains constant. The slope
Solution: l-
with respect to x. of the tangent line at P represents the rate at which z changeswith
dz = respect to x. Hence the partial deiivative $ is the slope of the
siny+ycosx
a*
tangent to.the curve of intersection APB at,the point P.
,^
Considering x as constant and differentiatinf I d

with respect to y, I
II
I
a ,
d z = .xcosy +
sinx
0y
EXAMPLE E. I f u : x z + y z z * x z , f i n d u x ' 5 '
<l Solution: Considering y and z as constants and differen.
t., tiating with respect to x,

<: ux= 2x* z

hr Considering x and, z as constants and differen.


tiating with respect to y, _)x

h, uy:
a

"
Tangent
/-' C o n s i d e r i n g x a n d y a s constants and differen.
\--., tiating with respect lo z, ,+
rEg
!z: Zyz* x v FIG. 8.1

fi,y, .The student who is not familiar with graphsof zurfacesshould rafer to any text
on analytic geometryfor a brief rwiew-
Visit For more Pdf's Books
Pdfbooksforum.com
and Integral Itrl'fr.rentiation 189
Diffbrential
v*-v^v-"-5
188

Find the equations of the tarlgent to the parabola


are
The equadons of the tangent at P z = x ' + 3y" ; y : 1 at the point (2,1, 7).

E(8.3) z-zo: so(x-xo) ,V-


The parabola which is the curue of intersection of
the surface z : x2 + 3yt and the plane y = 1
whereffio : value of # at P' is represented by the curve APB in Fig. 8.3 Its
vertex is at point A. Since y is constant, we dif-
., g^a arrxro t-! ot ferentiate partially the equation z : x2 + 3y'
ln Fig. 8.2, the curve CPD is the cunt:
of intersection
a point poyeq d with respect of x. Thus
the,planex-: xs. As
surface, : f(ri,'Vf *iiite x-remarns constant. Hc
the.curve cpD,.y ""q
and z uarv dz
zx
CPD at the ox
F t* the slope of the tangent to the curve
(ry
The equationsof the tangentat P are At (2,L,7),*u fr"u" = 2(2)=4.Hen""*o = 4,
ff
=mo(Y.-Yo), x t
E(8.4) n-zo Xo = 2, and zo = 7. Thenby E(8.3),the equations
of the tangent at P are
= value ot at P.
where mo #
z-'l=4(x -Z),y=1
or z=4x--1,V:1 t';'l

I Tangent

lbrl

f?:
thq

.. )x
\1

C
-i
FIG. 8.2
-:
Visit For more Pdf's Books
Pdfbooksforum.com
f inite Integral
222 ZZg

f(
( 1 ) Formula L2 can be extended to the sum of
,t)2. te" + 4)rdx+J.te*, + 24x+.16)dx
J
number of differentials.
(2) Formula I3 tells us that a constant may be =T. ry+ r6x+
c
the integral sign. (Note: You can not do this
ble.) 3x3 + L171,2
+ 16x+ C
(3) Formula 14 is used for finding. the integral
of a function. Note that it holds for any 'r'i3 * !)a*
n except n = - 1. Note further that if u : x,
fies to I(+. *)0,.=l(*-'
+c = rl*
,[""*: # I4x-'dx+
= '
f
$* 2Lntx1r c
EXAI\,iPLE 1. Evahrate + 3x2 + 6)dx
J ts*o
Solution: =-3+2rnlxl+C
: y* *Jr*,
+ I 6ro*
J.u* Bx2 I **J,
:sJxodx+sJ*
=H*c,.+*cr+6r
I 2dx+ 6
tht' following:
EXERCISE9.1 -

'-4x+5)dx

<q :xs+x3+6x*("1

:x5+x3+6x+C
+"2+
1) (3x + 4)dx
b-rrrll
r/i -- l)dx
72n, where* C : .1 * -enraluited
*
_"1. In Practice,
bq,
integral is siinply"z in the
bqrrir manner:
' t_,4x-- g
dx
f
\hr
/tu*-+Bx2+o)dx=Y*T* x'
,_ililt
=x5+x3+6r './;-- Zx rE)ox
rtqrr

Effii
E3ilil rConsturts cbn d(ways be combined into a dngle oonEtant.

Visit For more Pdf's Books


Pdfbooksforum.com
Differential and Inrh.finite Integral 226
224

f ' ff fd F ( u ) =
8'Jq#26" JJ
lf (u)du (3)

i;
( Alro, since u = h(x) (4)
F e. lffia" = h'(x)
J l'ltcn +
dx
(5)

" JG-{*Ao" lly the Chain Rule for differentiation,

4F(u) - gE(g). du (6)


dx du dx
9.3 Integration bY Substitution
ly ( l), (4) and (5), equation (6) becomes
Some integrals ean not be evaluated rea
------v- of the standard iltesralion .formulT:
catiqn dF(u'
[h(x)]h'(x)
evalubting suchr integralsleans heavily on whet d*':f
iethod 6f aubstitution.* Thig method _involverI
t

ble, say tiom x to another variable u' The pgrPq .rr dF(u):f[h(x)ttr'(x)dx (?)
i is to bring the problem to a foro "
""*'variabte
dard formula can be applied' This integration Intr.fratingboth sidesof (?) :
justified by the so called Chain Rule for in
briefly state below.
=/rrnt")Jh'(x)dx (8;
fot,",
, Let F(u) be a function whose (lrrrrparing (3) and (8), we seethat
tt"l = i1.t;. If u is a differentiable
* rf
rlupu=J rlntx)lh,(x)dx
then J .
= (
E(e.4) J tluyou [ lf [h(x)l h' rhrch is what we:wanted to prove.

Lgt us now proveE(9.4)' We areglventhat


$G1,,
F--
Let u: 3x * 4. Thendu:3dxor*a": dx.
SP:r(u) Then the given int€Fal becomes o

dF(u) = f(u)du
rr{.
Flp-' J te*+ 4\'dx:J "'. to"
'{::. Integratingboth sidesof (2)
: *J"'ou by IE

'b*='
l-gl
t
I
*othErmethodsofintegrationbysubstihrtionrviltbo

Visit For more Pdf's Books

t Pdfbooksforum.com
V 228 Differential and In
229

Solution: Perhapsto a beginner, this problem Flvaluate -6x*

J
f-r*,
-;=T-ox 4
quite difficult.: [t seemsnot easy to J
which expression should be equated to
time, we need that little "trick" we llere f(x) _ 2x2 - 6x * 4 and g(x) :
x _ B.
above. To do the trick, all we need ()urryine out the indicated

f(
algebraic maniPulations. Thus
2x2-6x+4
*lT-
_ ozx
= _ _+
divlsion, .u get
4
+x-=- nil i
+ 1 .Ix = | x(4x2 + 1) (4x2 +
| (a*t + x) t/ 4*
Jf.3J

ti
: l4*'-+1)Txdx (letu= where Q(x) = 2x and R(x) - {,
J'
(nf= 'l'herefore,

"
=1.{1*.'+1)+. 11
8-+c 2
f z x ' x?--36 x + 4 d x : f r-r4ou*a*-fJ ; 5 o *+ r.
J J
=(k1+1.)*
20
*" :x2+4ln
l*-gl +c
Consider this time the problem of evaluating the
of the rational fraetion -
]
rel
e(x)
where the degree of f(x) ) delpee of g(x'). To evaluate
EXERCISE9.2

ol t lre following indefinite integrals.

rlx
/
*
Dt must first carry out the indicated division until the remai
lower degtee than the denominator. That is,
lf
Irh :iili
\l
lrl' (6xt + 4)dx
#t h1
r
'ffir
t'
F
h
ON
tsttl
ffi:Q(x).ffi l lodx J#
li
l
fr
l1 where Q(x) = quotient
lrri
b R(x) -- remainder of lower deglee than g(x)

C brr
tstl
Therefore, we have l[
r- brr
fBdx=[h,.,.#-e]*
) s(xl J L-'-' s(*I
1
Ir
eltl

G, bl
Jttt =f 't*)dxi'Jffi* i11
ittt
Itatt'

J
illi
Visit For more Pdf's Books
Pdfbooksforum.com l
230 Differential and 231

f x2dx
", t' ${0,.
Jffi
f
| y'ln4x dx !.'- 2x. o*
8. x"2+]-;
2+L
J ---
It t 4x t
9. fdx -b-
-L
ox
J xln2x t'

10. f "'*u
JVF
f ,' of Ttigonometric Functions
11 J#T
l'rd formulas for evaluating the integrars of the six
functions are given below. The first two can be
dtff*rentiation and the remaining four may be proved

r
Jsinudu--cosu*C {
r
cosudu- sinu * C
I
:- rnlcosul
+c
JItu'uau
.
cotudu= ln
-/f fsinul+ C
,
: ln
secudu
J fr""r + tanul+ C
,,r.udu=- lnlcscu* cotul + C
J

Since d(-cosu + C) - sinudu, then we have


vr.rifiedthat T1 is correct.

tt"t
I t.n.,du- [ 0,,
.t J cosu
Fg
J,nr
F r-[l

r Visit For more Pdf's Books


Pdfbooksforum.com
234 Differentiql and Integral 'l'heIndefinite
Integral

EXEBCISE 9.8 f 4rirr'xcos2x


'Io'
Evaluate each of the following: J m;;;-d*
f fr
1. I secSxtanbxdx t6. I s*
J J tanSx
2.(dx
l-
J Slnxcosx fdx
!1 |
J sinSxtanSx
3. f rirr** cosx
J - o x sln- x
f e.6 Integration of Exponential Functions
4. | *"' (4x - 3)dx
J The foltowing formulas for evaluatingthe integralsof expo-
5.(dx ncntialfunctions qan be provedby differentiation.
J'lffi
81. f""a,r=etr+C
6. fer -laudu:#+c,
J l-cosx Ez. a) 0,h+ 1-
7. ( :
---
dx
J ps63
"otj,* TXAMPLE1. Evaluate
f "a*a*
8.
f Solution: Here u = 4x and du = 4dx. Hence nf :*. Ay nf ,
I cos4x 4
rf - 6sin2x
1
2t;t frr
-
J "n**=|"4*+c
\Lr) f
g. I tr + hnx)zdx
-El Jr
10. | *'cos4x3dx
Ix AMPLE2. Evaluate
2€r
--l J
J43xdx
11. f cos6xdx Solution: This takes the form au with a - 4 and u = Bx.
DEI a
n

J cos23x Hencedu : Bdx andrrf: . By EZ,


r t
L2. I sin2xsecxdx' ( i
rtll o r ,r3x
J
f.fu
f4dxax= +c
,al 13. ( sin2xdx

IQr
J zri"-"*'- : axB
+ c
f 31,14
Egl L4. | (cotx * tanx)2 dx
J 43x
2 Jr : - _ E . _ ^
' \-/
D-u, In64
Visit For more Pdf's Books
Pdfbooksforum.com
936 Differential and Integral Irnite Integfal 237

r
EXERCISE 9.4 sech'?udu: tanhu + C
J
f
Evaluate the following: csch2udu =-cothu + C
I
"f
1 f a* ,tanhudu=-sechu * C
r' .r-.."n
J
"2* f ,,schucothudu= - cschu* C
f
Jn
!,. + t12a:r
J ,"t* tanfrudu= ln lcoshul+ C
/- / ?
g.
f "sin4x"os4xdx cothudu- ln lsinhul+ C
I
4. f"*=+e-** cosh(4x+ 3)dx:f sinh(4x + 3) + c by Hl
Jex J
f
5. dx =f r"lcosrrx'[+ c
J,.,Fa* /r*rr*r*2
byH?
6.
Iro** +'

7. dx t';i

fa,-2r EXERCISE9.6
f\

3. {42x dx the following:


J
h (3x - 1)dx
9.
f e*z*a* 4x * sinh2x)dx
10.
Jzx
|^{u* t(1 -x').xdx

9.6 Integration of Hyperbolie Functions *xtanh*xdx


>r.a
The following formulas are used for evaluating the in rhz(lgr.Idx
of hyperbotic functions. Formulas Hl to 'H6 may be x
>--,, by differentiation. For example, FI1 is conectsinced(sinhu+
trth(1 - 2x)dx
coshudu. The student may give the proof of H7 and H8.

lb- r:'h txcoth{xdx


Hl. *du = sinhu * C 4x
-g /.ort dx
h2x
H2. sinfrudu : coshu * C
-=' /
Visit For more Pdf's Books
Pdfbooksforum.com
fntr.gration 247
246 Uifferential and

(or two sines)can be reduced to an integral EXERCISE10.1


rence)of two cosines.That is
of the following:
Pl. = sin(u+ v)
frrrnucosvdx
II
P2'. =/ +
/z"orrr"osudx [ coslu v)

P3. - v]
cos(u il)cos(x+ 5)dx
J I
frrr^usinvdx
lr
The right member of Pl is then '|4..t'x(lx
and those of P2 and PB by T2. Considerthe.
EXAMPLE 1. Evaluate
I
Solution: We have the product of two
2r)cos(x* n)dx

h )sin(2x + 3)dx

and v = 2x. Hence, we shall ure nl l xdx

j"'u'cos2xdx { lcos(x+ 4,)ax


: :f;::'.:::,., t ' : i l

-{ )a"
r { ysin12*
=+ (ltitta*+dsin{r
.
: ,{ sin8x + i sin4x i [;eos(x+ 4)dxt
EXAMPLE 2. Evaluate /esin5
J
Solution: We have the product of a rlno
u : 5x and v = 4x. Hence,wt
by usingPl.
rf : +J
SsinSxcos4xdx 2sinSxeos4xdx
/
= +f$nex*sinx)dr
_ + (- *""t9x - .oo)+
-
:- + cosgx -i cosx* Q

Visit For more Pdf's Books


Pdfbooksforum.com
r 252 Differential and Integral of lntegration 253

1\ of Tangents and Secants


- - -L[ -/3x - sin2x1:sin4x) + C
4\2 8t
k,r the trigonometricintegralof the form
3x sin2x * sin4x +C f't"r,*urecnvdx
432 J
I 1, we evaluate the integral by T9 (see Section 9.4).
number and n = 2, we evaluatethe integral by tle or-
EXERCISE10.2
hod of substitution usedin Section 9.3. For example,
u = tan x, we canshowthat
Evraluatbeachof the following: , xsec2xdx : tuto,r *
c
t. f sint xeosaxdx 4
JnI
2. f ,sin3Zx r,,6ffi ax rection, we shall cgnsider the following cases:
-f
3. sina3xcos3Sxdx When tn is any number and n is a positiue utrm
J integer grcater than 2, we may write
4. f
""fu a*
J sinsx tanmvsecnv: (tanmvsect-2v) r""2n
{.,_I

xdx
5. f sir,o'*"os2 and then use the identity
J-
6. / tto xcosaxdx sec2v=1+tan2v
rf,
7. (!ffi+ cosx)2dx to reduce the grven integral to the form
J
f f
t. * cos4x)2dx
J_(t | (sum of powers of tanv)sec2vdx
J
9. cos2x)'dx 'tanu.
which is now integrable by 14 and with o. =
- f/tri"a**
f
tO. sin2Txdx I . Evpluate
J I t"nt xsecaxdx
J
11. f 4xdx rll:
t "or'
b-,- u. [
f
rx.t:axdx : I tant xseczxsec2xdx
Fr-rn
J. "osTxdx J.
I
13. I sint 2xdx = x (1 + tan2 x)eec2xdx
J- | t*t
\,- r J
f
Iq L4. I sint x cossxdx
J
: | (tant x * tansx)sec2xdx
J
f
-* ,U. sin?xeos3x dx _ tgrn *_ + jg!,t * +
J c
Visit For more Pdf's Books
Pdfbooksforum.com
r
I
Differential and

EXERCISE 10.3
thods of Integration

,4 Powers of Cotangents and Cosecants


Evaluatethe following:
'l'he technique involved in evaluating the-integral

l
1. tun'2x$eca2xdx; r
f
ur
,2. I cotmvcscnvdx
*"t xseca
xdx J
J r(, v is a differentiable function of x, is similar to thai for.
f
3. xdx
Vt""* sec6 trcl rurtingthe integfal
-rI
4. tanTxax f"turr-*ectrvdx.
J J
b. I +*0" ' identity esc2v = 1 + cot2v or cot2v = csc2v - 1 is used to
-f
uce the original expression into an .integrable, fonn. It also
6. *"t 4xdx of three possible easesand it is left to the student to write
|
-ftr""* n the procedure for evaluating each case.
7. + tanx)2
dx
rf
- sec4x)2 EXERCISE10.4
8. dx
J tt
'Jm'dx I
luateeachof the following:

xdx
--:t

f -"otoxcsca
10. f*rr'xsec3/2xdx I xdx
J J "ottxcsc3
r1. f-e"'.l_o* I
-| - f "ot'4xdx
1/secx r
a - 1)'dx
f,_ J-(csc'x
L2. VsdCExsec2SxtanSxdx
J o
J,tffiicsca Bxdx
'rt-*
I coss2xdx
f
.l sinE2x

r-"
L,rr-r,,
t
./
t"r"+* * cot4x)2dx

-q,' c I
t't& o*
/ t:ot6 x
FP' ,uIi,. Visit For more Pdf's Books
I

Pdfbooksforum.com

Potrebbero piacerti anche